MSC Maths Optional Paper V

You might also like

Download as pdf or txt
Download as pdf or txt
You are on page 1of 185

MECHANICS

UNIT I Mechanical Systems : The Mechanical System - Gen-


eralized Coordinates - Constraints - Virtual work - Energy and Mo-
mentum. Chapter 1 : Sections 1.1 to 1.5.
UNIT II Lagranges Equations : Derivation of Lagranges equa-
tions - Examples - Integrals of motion. Chapter 2 : Sections 2.1 to 2.3
(omit sec 2.4)
UNIT III Hamiltons Equations : Hamiltons Principle - Hamil-
tons Equation - Other variational Principle. Chapter 4: Sections 4.1
to 4.3 omit Section 4.4.
UNIT IV Hamilton - Jacobi Theory : Hamilton Principle func-
tion - Hamilton - Jacobi Equation - Separability. Chapter 5 : Sections
5.1 to 5.3
UNIT V Canonical Transformation : Dierential forms and gen-
erating functions - Special Transformations - Lagrange and Poisson
brackets. Chapter 6 : Sections 6.1, 6.2 and 6.3 (omit sections 6.4, 6.5
and 6.6)
Contents and Treatment as in : D. Greenwood. classical Dynam-
ics. Prentices, Prentice Hall of India, New Delhi, 1985.
Reference :
1. H. Goldstein, Classical Mechanics, (2nd Editin) narosa Publish-
ing House, New Delhi.
2. N.C. Rane and P.S.C. Joag, Classical Mechanics, tata McGraw
Hill, 1991.
3. J.L. Synge and B.A. Grith, Principles of Mechanics (3rd Edi-
tion) MCGraw Hill Book Co New yuk, 1970.
1
UNIT I
Introduction :
Mechanics forms the basis of physics it contributes to our knowl-
edge of people working of nature and universe classical mechanics deals
with the description of actual or possible motion of points like as well
as extended bodies.
The motion of celestial bodies as well as man made objects such as
space problems. Satellites etc., are elds. The various classical dynam-
ics, Hamilton mechanics, provides most modern research in frontial ar-
eas. Particularly relation between symmetric property or conservation
law.
Application :
Mechanics is the study of motion of physical bodies. The motion
of celestial bodies planets, stars etc., path of an artillery shell or of a
space satellites send from a earth to a planet are among its problems.
Classical mechanics denotes part of Mechanics. Classical dynamics
will also be interpreted to include the type of mechanical assising to
include the type of Mechanical assising out of the special theory of
relativity. Mechanics of a system of particles :
When a Mechanical system consists of two (or) more particles we
must distinguish between the external forces acted upon the particles
of the system forces not belongs to the system. The internal forces
assising on account of interaction between themselves.
Mechanics of a particles :
Let r be a radius of the particle from some given origin and v its
the vector velocity then
v =
dr
dt
.
The linear momentum p of the particle is dened as the product
of particle mass and its velocity. In consequence of interaction with
external objects and eld the particle may experience forces of various
types.
2
For example, gravitation or electro dynamics.
The Mechanic of particle is contained in Newton second law mo-
tion.
Equation of Motion :
The motion of particle is described by the dierential equation
F a

F = m

a
= m
_
d

v
dt
_
=
d
dt
(m

v ) =
d
dt
(p)

F =
dp
dt
F =
d
dt
(mv)
= m
d
dt
_
dr
dt
_
= m
d
2
r
dt
2
F = m
d
2
r
dt
2
Thus the equation of motion is a dierential equation of second
order.
Equation of motion of System :
The equation of motion of a system of N particles is written as
m
i

r =

F
i
+

R
i
(i = 1, 2, 3, , N)
where m
i
is the mass of the i
th
particle,

F
i
is the applied force
and

R
i
the constraint force.
Generalized coordinates :
To described the conguration of the system we select the smallest
possible number of variables which dene conguraiton of the system.
These are called the generalized co-ordinates of the system.
3
Conguration :
A set of generalized co-ordinates is any set of co-oirdinates which
described the conguration.
Notation for generalized Co-ordinates :
Generalized co-ordinates are q
1
, q
2
, , q
n
(q
i
, i = 1, 2, , n) the
symbols q
1
, q
2
, , q
n
corresponding to the co-ordinates that we choose
to describe the motion.
1. When the particle move in a plane it described by cartesian co-
ordinates (x, y) or polar co-ordinates and soon
q
1
= x
q
2
= y
or
q
1
= r
q
2
=
2. Some spherical symmetry the sherical co-ordinates is
q
1
= r =
_
x
2
+ y
2
q
2
= = cot
1
_
z
_
x
2
+ y
2
_
q
3
= = tan
1
_
y
x
_
Denitions :
1. Newton fundamental equation
2. System of particle
3. Degree of freedom
4. Generalized Co-ordinates
5. Conguration space
6. Holonomic constraints
7. Non polynomic constraints
4
8. Scleronomic constraints
9. Rheonomic constraints
10. Clarnomic system
11. Realnomic system
12. Polyateral constraints
13. Unilateral constraints
Degree of freedom :
The number of degree of freedom is equal to the number of co-
ordinates minus the number of independent equation of constraints.
(or)
If the conguration of a system of a N particles described using de-
gree N cartesian relation these co-ordinates then the degrees of fredom
equals 3N .
Example :
Let 3 particles connected by the rigid rods to form a triangular
body with the particles are at the corners. There particles are xed
by a cartesian co-ordinates each rigid rod is represonted mathemati-
cally by indepenent constraints equation. All together there are three
constraints. Hence degree of freedom equals 9 3 = 6.
Generalized co-ordinates :
A set of parameters used to represent the conguration of a system
without ambiguity are called Generalized co-ordinates.
Conguration Space :
Let n be the degree of freedom of a system of N - particles. Let
its congurations be specied by n - generalized co-ordinates.
The n - dimensional space representing these Co-ordinates as a
single point q (q
1
, q
2
, , q
n
) is called a conguation space.
Holonomic constraints :
Let the conguration of a system be specied by n - generalized
Co-ordinates (q
1
, q
2
, , q
n
).
5
Let us assume that ethere are k independent equation of constraints
of the form

j
(q
1
, q
2
, , q
n
, t) = 0, j = 1, 2, 3, , k.
A constraints which can be expressed in this fashion is known as a
Holonomic constraints.
Holonomic system :
A system is said to be Holonomic if its constraints are always Holo-
momic.
Example :
Consider the motion of 2 particles connected by a rigid rod of length
in xy - plane constraint equation is given by
(x
2
x
1
)
2
+ (y
2
y
1
)
2
=
2
.
Scleronomic constraints :
The constraints is said to be scleronomic if time t does not appear
explicity.
Example:
Consider the motion of two particle in the xy plane they are con-
nectd by a rigid rod of length . The corresponding constraint equation
is

2
= (x
2
x
1
)
2
+ (y
2
y
1
)
2
.
where x
1
, x
2
, y
1
, y
2
are cartesian co-ordinates of 2 particles.
The equation does not contain the time t explicity.
Scleronomic system :
A system is said to be scleronomic if its constraints are scleronomic.
Rhenomic system :
A system is said to be Rhenomic if its constraints are Rhenomic.
Rhenomic constraints :
If the length have been generalized as an explicity function of
time then the constraint is Rhenomic. OR Constraint relation depend
6
explicity on time.
Non - Holonomic constraints :
Let a system has m contraints which are non - integrable and dif-
ferential expression is of the form
n

i=1
a
ji
dq
i
+ a
jt
dt = 0, j = 1, 2, 3, , m
where a

s is a function of the q

s and t.
This type of constraints are known as non- Holom Holkonomic
constraints.
Bilateral constraints :
If any conguration of the system for every small allowable dis-
placement in any xed time. The negative displacement is also al-
lowable then the constraints are called bilateral constraint. They are
expressed as equalities.
Unilateral constraint :
In any conguration of the system for every small allowable dis-
placement in any xed time either a positive or negative displacement
is allowable then the constraints are called unilateral constraints. They
are experessed as in equlities.
f (q
1
, q
2
, , q
n
, t) 0 (unilateralconstraints)
f (q
1
, q
2
, , q
n
, t) = 0 (Bilateral)
Example :
Let a free from particle be contained in a xed hollowsphere of
radius r if its centre at the origin of a cartesian co-ordinates system.
If (x, y, z) are the co-ordinates of the particle then x
2
+ y
2
+ z
2
0
where r is trhe radius.
Virtural displacement :
Let a system of N particles is given by 3N cartesian co-ordinates
x
1
, x
2
, , x
3N
with respect to some intertial frame adn may be subject
7
to the contraints. At any given time let us assume that the co-ordinates
have small innite displacement x
1
, x
2
, , x
3N
.
They are assumed to occure without passing of tiem and donot
necessary conform to the constraints. These small change Sk in the
conguration is known as Virtual displacement.
Virtual work :
Consider a system of N particles. Let the conguration to repre-
sented by the cartesian co-ordinates x
1
, x
2
, , x
3N
.
Let f
1
, f
2
, , f
3N
are the component forces applied at the corre-
sponding
Co-ordinates in a positive sense then the virtual workdone (W)
of these forces for a virtual displacement x is given by
W =
3N

j=1

F
j

x
j
Note :
In a vector form W =
N

i=1

F
i

r
i
where

F
i
is the applied force of
i
th
particle

r
i
is the positive vector.
The forces are assumed to be constant through out the virtual
displacement.
Workless constraints :
A workless constraints is any bilateral constraint such that the
virtual work done by the corresponding constraint force is zero for any
virtual displacement, which is consistent with the constraint.
Examples :
1. Sliding motion on a frictionless susface.
2. Rolling contact without slipping.
Book work : The virtual displacement is not ingeneral a possible
real displacement.
Proof : In the virtual displacement, the moving constraints are
8
assumen to be slopped.
Let the system have k holonomic constraints.

j
(q
1
, q
2
, , q
3N
, t) = 0, j = 1, 2, 3, , k. (1)
Let us take the totla dierential of and obtain
d
j
=

j
q
1
dq
1
+

j
q
2
dq
2
+ +

j
q
3N
dq
3N
+

j
t
dt = 0.
Suppose the system is subject to k holonomic constraints

j
(x
1
, x
2
, , x
3N
, t) = 0, j = 1, 2, 3, , k.
d
j
=

j
x
1
dx
1
+

j
x
2
dx + +

j
x
3N
dx
3N
+

j
t
dt = 0.
Virtual displacement
dx
i
= x
i
, dt = 0 (2)
d
j
=
3N

i=1

j
x
i
dx
i
+

j
t
dt = 0, j = 1, 2, 3, , k.
Case (i) :
Suppose te system has non-holonomic constraint. Then
3N

i=1
a
ji
dx
i
+ a
jt
dt = 0, j = 1, 2, 3, , k.
Virtual displacement dt = 0, dx
i
= x
i
n

i=1
a
ji
x
i
= 0 (j = 1, 2, 3, , m)
Form (1) and (2) shows that any holonomic constraint must also
9
be scleronomic constraint
i.e.,

j
t
= 0, j = 1, 2, 3, , k
Suppose non-holonomic constraint a
j
= 0 . These constraint are
not ment in the general case.
Hence a virtual displacement is not possible as a real displacement.
State and prove the principle of virtual work :
Statement:
The necessary and sucient condition for the static equilibrium of an
initially mottionless sceleronomic which is subject of the system in to
the virtual workdone for an arbitrary virtual displacement satisfying
the constraint is zero.
Proof :
Case (i)
Let us assume that we are given an sclernomic system of N particles.
Let us assume that their system is in static equilibirium. Then for
each particle

F
i
+

R
i
= 0
where

F
i
is the external force.

R
i
is the constraint force acting on i
th
particle is
_

F
i
+

R
i
_

r
i
= W, i = 1, 2, 3, , n
where

r
i
is the position vector of i
th
particle.
Virtual workdone by all the force in moving through an arbitrary
vitrual displacement consistant with the constraint is zero.
N

i=1
_

F
i
+

R
i
_

r
i
= 0
N

i=1

F
i

r
i
+
N

i=1

R
i

r
i
= 0
10
constraints are workless, then
N

i=1

R
i

r
i
= 0
N

i=1

F
i

r
i
= 0
i.e., W = 0
workdone = 0, i.e., the virual workdone by the applied force is zero.
Case (ii) :
Let us assume that the same system of particles are initially mo-
tionless. But it is not in static equlibirium.
The motion is s to C we can always choose a virtual displacement
in the direction of actual motion at each particle.
In this case, the virtual work is positive
N

i=1

F
i

r
i
+
N

i=1

R
i

r
i
> 0
W =
N

i=1

F
i

r
i
> 0,

F
i
s = 0
W > 0
If the system is not in equlibrium then the virtual workdone is not
zero.
1. Show that virtual workdowne by the constraint forces
on a system of two particles connected by the rigid massless
rod is zero.
Solution :

R
i
,

R
i
are the constraint force along the rod on the 2 particles.
They are opoosite in sense with equal magnitude.

e
r
is the unit vector along the rod.

r
1
,

r
2
are the virtual dis-
placement at the ends virtual workdone by a constraint force is given
11
by
W
C
=

R
1

r
1
+

R
2
r
2

R
1
= R
1
(

e
r
) = R

e
r

R
2
= R
2
(

e
r
) = R
1
virtual displacement along the rod all equal

r
1
e
r
=

r
2
e
r
W
C
= R
1

e
r

r
1
+ R
2

e
r

r
2
= (R
2
R
1
)

e
r
r
1
W
C
= 0 ( R
2
= R
1
equal magnitude) .
Example for bilateral constraints :
2. Given an equation of scleronomic with workless constraints
(bilateral) to which the principle of the virtual work can be
applied (or)
Two frictionless blocks of equal mass m all connected by
the massless rigid rod using x
1
and x
2
as co-ordinates. Solve
for the force F
2
. If the system is in static equlibirium.
Solution :
R
1
and R
2
are the external constraint acting at the wall and oor
repectively.
The applied forces are the gravitational force acting in the blocks
and the external forces F
2
system is in static equlibirium.
Virtual workdone by external force (F) = 0
Workdone by constraint force (R) = 0
We have
mg(x
1
) +F
2
(x
2
) = 0. (1)
The virtual displacement along the rod at ends are equal.
x
1
sin = x
2
cos (2)
12
Multiply (1) by sin ,
mg sin x
1
+ F
2
sin x
2
= 0
mgx
2
cos + F
2
sin x
2
= 0
(mg cos + F
2
sin ) x
2
= 0.
But displacement x
2
= 0.
A frictionless system which is constrained to move in the vertical
plane.
mg cos + F
2
sin = 0
F
2
sin = mg cos
F
2
= mg cot
3. Using a suitable example, show how the concept of
virtual work can be applied to system with unilateral con-
straints.
Solution :
Let us assume a cube of mass is placed between two frictionless
mutually perpenticular plane.
Assume that the motion is an vertical plane. Le x
1
= x
2
= 0 at the
equilibirium position then unilateral constraints are x
1
0, x
2
0.
The only applied force is the weight mg. It has components F
1
, F
2
in
x
1
and x
2
direction.
m
45
0
45
0
g
x
1
R
1
R
2
x
2
When in equilibuim R
1
= F
1
and R
2
= F
2
W
C
=
mg

2
(x
1
+ x
2
) > 0
13
F
1
= F
2
=
mg

2
Virtual workdone by applied forces is
W =
mg

2
(x
1
+
2
) 0
Thus the virtual W 0 for any virtual displacement consistent
with unilateral constraints.
To nd workdone by constraint forces
R
1
= R
2
=
mg

2
Virtual workdone by constraint forces
W
C
= R
1
x
1
+ R
2
x
2
=
mg

2
(x
1
+ x
2
) 0.
Hence for a system with unilateral constraints
1. Workdone by external force is ve.
2. Workdone by constraint force is + ve.
D Alemberts Principle :
Let the system has N particles. Consider on i
th
particle to the
system m
i
is the mass of the particle.

r
i
is the position vector of particle.

F
i
is applied force of particle.
Let

R
i
be constraint force of particle.
Let

r
i
be the acceleration of i
th
particle relative to an inertial
frame.

m
i

r
i
is the intertial force acting on the i
th
particle.
Now we have the equation

F
i
+

R
i

m
i

r
i
= 0, i = 1, 2, 3, , N. (1)
(The sum of all the force real and inertial acting on each particle of a
system is zero)
This result is known as D Alemberts principle.
14
Note :

F
i
and

R
i
together are called as real forces.
Lagranges form of D Alembrts Principle :
Let us assume that the system has N particles. Let m
i
be the mass
of the i
th
particle frame.

F
i
is the applied forced and

R
i
is the constraint force on the i
th
particle.

m
i

r
i
is inertial force acting on the particle. Now we have the
equation

F
i
+

R
i

m
i

r
i
= 0 (i = 1, 2, 3, , N) .
Now the virtual workdone by the system is
N

i=1
_

F
i
+

R
i

m
i

r
i
_

r
i
= 0

R
i
is the workless constraint forces
N

i=1

R
i

r
i
= 0
N

i=1
_

F
i

m
i

r
i
_

r
i
= 0.
This equation is the Lagranges form of D Alemberts principle.
1. Obtain the dierential equation of the given pendulum
(sphericla pendulum) or A particle of mass m is subspended
by a massless wire of length r = a +bcos t(a > b > 0) to form
a sphorical pendulum. Find the equation of motion.
Solution :
Let us assume that the spherical co-ordinates and . is mea-
sured from the upward vertical. is the angle between a verticla
plane through a supporting point and a vertical plane containing the
pendulum.
15
O
r
m
e
f e
q
e
r
g
q
f
The acceleration of a particle whose spherical co-ordinates are
(r, , ) is as follows

r =
_

r r

2
r

sin
2

e
r
+
_
r

+ 2

2
sin cos
_

+
_
r

sin + 2

sin + 2r

cos
_

where

e

e
r
,

are unit vectors forming an orthogonal triad.


A virtual displacement constraint with the constraint is,

r = r

+ r sin

.
The applied gravitational force is,

F = mg cos
_
180
0

e
r
+ mg sin
_
180
0

F = mg cos

e
r
+ mg sin

we have
_

F m

r
_

r = 0 (By D Alemberts principle)


16
(mg cos

e
r
+ mg sin

) m
__

r r

2
r

sin
2

e
r
_
+
_
r

+ 2

2
sin cos
_

+
__
r

sin + 2

sin + 2r

cos
_

_
r

+ r sin

= 0
Taking dot product,
mgr sin mr
_
r

+ 2

2
sin cos
_

m
_
r

sin + 2

sin + 2r

cos
_
r sin = 0
and are independent Virtual displacements.
co-eceient are equal to zero respectively
mr
_
g sin
_
r

+ 2

sin cos
__
= 0 (1)
Since mr sin = 0
sin mr
_
r

sin + 2

sin + 2r

cos
_
= 0 (2)
r = a + b cos t
r = b sin t
(1) (a + b cos t)

sin 2b sin t

(a + b cos t)

2
sin cos g sin = 0
(2) (a + b cos t)

sin 2b sin t sin

+2 (a + b cos t)

cos = 0.
These are dierential equation of the motion.
Generalized forces :
Let us assume that a given set of force F
1
, F
2
, , , F
3N
is applied
to system of N particles.
17
The virtual workdone by these force is
W =
3N

j=1
F
j
x
j
(1)
Let us suppose that the 3N ordinary constraint co-ordinates are
related with the n - generalized co-ordinates q
1
, q
2
, , q
n
.
Let these are related by the relations
x
j
= x
j
(q
1
, q
2
, , q
n
, t), j = 1, 2, 3, , 3N.
Using displacement is virtual workdone we get,
dx
j
=
x
j
q
1
dq
1
+
x
j
q
2
dq
2
+ +
x
j
q
n
dq
n
+
x
j
t
dt
x
j
=
x
j
q
1
q
1
+
x
j
q
2
q
2
+ +
x
j
q
n
q
n
x
j
=
n

i=1
x
j
q
i
q
i
(j = 1, 2, , 3N)
(1) W =
3N

j=1
F
j
_
n

i=1
x
j
q
i
q
i
_
=
n

i=1
_
3N

j=1
F
j
x
j
q
i
_
q
i
3N

j=1
F
j
x
j
q
i
is called the generalized force.
W =
n

i=1
Q
i
q
i
.
Principle of work and kinetic evergy :
Statement:
The increase in K.E. of a particle as its moves from one arbitary point
to another point is equal to the workdone by the force acting on the
particle during the given intervals (or) chang in K.E. = workdone.
Proof:
Kinetic Energy is given by T =
1
2
mv
2
v is the velocity of particle and
18
m is the mass of the particle.
Let W be the workdone by the force

F in moving particle from A
to B is
W =
B
_
A

F d

r
=
B
_
A
m

r d

r ,

F = m

r
= m
B
_
A

r d

r
W =
1
2
m
B
_
A
d
dt
_

r
_
dt
=
1
2
m
B
_
A
d
_

r
_
2
=
m
2
B
_
A
d
_

r
_
2
=
m
2
B
_
A
d
_
v
2
_
=
m
2
_
v
2
_
B
A
=
m
2
_
v
2
B
v
2
A
_
=
1
2
mv
2
B

1
2
mv
2
A
W = T
B
T
A
Book work : State and prove the principle of conservation of
energy.
Statement: Let the only force acting on the given particle be conser-
vative.
Proof : Let

F be the conservative force acting on the particle at the
19
point (x, y, z).
m is the mass of the particle.
T is the K.E. of the particle.
W is the workdone of the particle, from A to B
W = V
A
V
B
= T
A
T
B
Since the points A and B are arbitrary.
T
A
+V
A
= T
B
+T
B
= E of the energy is conservative. We conclude
that the total mechanical evergy E remains constant during the motion
of the particle.
This is the principle of conservation of evergy.
Book work :
An equilibrium conguration at a conservative holonomic system
with workless xed constraints must occur at a position where the P.E.
has a stationary value.
Proof :
consider a system of N particles whose applied forces are conser-
vative and derived from the P.E. function V (x
1
, x
2
, , x
3N
).
The virtual workdone by the forces
W =
3N

j=1
Fx
j
x
j
But Fx
j
=
V
x
j
Workdone W =
3N

j=1
_
V
q
i
_
x
j
= V.
This is the rst variation of the P.E.
By principle of virtual work the necessary condition for the static
equilibrium of the system is W = 0, V = 0.
V
x
j
= 0, j = 1, 2, 3, , 3N.
20
Let us assume the P.E is interms of generalized co-ordinates
V = V (q
1
, q
2
, , q
n
)
V =
n

i=1
V
x
i
(q
i
) , q

s are independent
We conclude that an equilibrium conguration of a conservative
holonomic system with workless xed constraints must occur at a po-
sition.
Where the P.E has a stationary value. Consider the stability of
this system at position of static equilibrium.
V = 0

V
q
i
= 0, i = 1, 2, 3, , n.
These condition imply that the P.E. is at a stationary value system.
Expanding the P.E. function V (q
1
, q
2
, , q
n
) about the reference
value V
0
, we have
V = V
0
+
_
V
q
1
_
0
q
1
+
_
V
q
2
_
0
q
2
+
+
1
2
_

2
V
q
2
1
_
(q
1
)
2
+ +
_

2
V
q
1
q
2
_
(q
1
q
2
) +
V is the change in P.E. from its value of equation position.
V = V V
0
=
1
2
_

2
V
q
2
1
_
_

2
q
1
_
+
_

2
V
q
1
q
2
_
(q
1
q
2
) +
1
2
_

2
V
q
2
2
_
_

2
q
2
_
+
where a zero subscript on a function implices that it is to be evalu-
ated at the reference value of the qs. The qs represent innitesimal
change from this reference conguration.
(i) If V > 0 The equilibrium is stable
(ii) If V < 0 The equilibrium is unstable
21
(iii) If V = 0 The equilibrium is nutral.
K
..
onigs Theorem :
The total K.E. of a system is equal to the sum of
(i) The K.E. due to a particle having a mass equal to the total mass
of the system and moving with the velocity of the centre of mass
and
(ii) the K.E. due to the motion the system relative to its centre of
mass.
Find the K.E. of the system of particle.
Proof :
Consider a system of N particles.
Let

r
i
be the position vector of the i
th
particle (whose mass is no)
relative to a point O xed in an inertial frame.
j
m
i
m
y
O
i
r
r
i
m
. c m
C
r
r
C
r
r
x
z
Let

r
C
be the position vector of the mass centre om.
Let m
i
be the mass of the system.
Let T be the K.E. of the system.
T =
N

i=1
1
2
m
i

r
2

r =

r
C
+

i
22

r
2
=

r
2
C
+

i
2
+ 2

r
C

i
T =
1
2
_
N

i=1
m
i
_

r
2
C
+
1
2
N

i=1
m
i

i
2
+
N

i=1
m
i
_

r
C

i
_
=
1
2

r
2
C
N

i=1
m
i
+
1
2
N

i=1
m
i

i
2
+

r
C
N

i=1
m
i

i
=
1
2

r
2
C
N

i=1
m
i
+
1
2
N

i=1
m
i

i
2
_

i=1
m
i

i
= 0
_
T =
1
2
m

r
2
C
+
1
2
N

i=1
m
i

i
2
T = K.E of mass centre + K.E. relative to the centre of mass.
K.E. of a particle of mass m with vector of c.m.(centre of mass)
Book work :
Kinetic Energy of a system of a particle using an arbitrary reference
point.
Proof :
Consider a system of N - particle. Let

r
i
be the position vector
of an i
th
particle (mass m) relative to a point O xed in an inertial
frame.
Let P be an arbitrary point.

r
C
is the position vector of mass
centre c.m.
k
m
y
O
i
r
r
i
m . c m
P
C
r
r
i
r
r
C
r
r
z
23

c
is the position vector of mass centre relative to the point P.
Let T be the kinetic energy of the system.
T =
N

i=1
1
2
m
i

r
2

r =

r

+

r
2
i
=
_

r

+

i
_
2
=

r
2

i
2
+ 2

i
T =
1
2
N

i=1
m
i

r
2

+
1
2
N

i=1
m
i

i
2
+
N

i=1
m
i

i
=
1
2
N

i=1
m
i

i
2
+
1
2

r
2

m +

r

N

i=1
m
i

i
T =
1
2
m

r
2

+
1
2
N

i=1
m
i

i
2
+

r

m

C
_

C
=
1
m

m
i

C
_
Hence the total K.E. is the sum of this 3 parts.
1. The K.E. due to the particle having a mass and moving with
reference point P.
2. The K.E. of the system due to its motion relative to P.
3. The sclar product of the velocity of the reference point and the
linear momentum of the system relative to a reference point.
Note :

i
=
C
+
j

i
=

C
+

i
=

C
+

j
But

m
j
= 0,

i
=

C
24
Kinetic Energy of rigid body :
Take CM as the origin of vectors. Let dV be a small volume element
of a rigid body with density.
Each element of the rigid body is having in general translational
and rotational velocity.
We can assume that the rigid body is having instantaneous axis of
rotaion. The typical volume element dV can be chosen so small.
ie, rotational K.E is negligibe compared with its translational K.E
Each element of a rigid body can be considered as a particle of
innite small mass.
m = V
Hence the limiting case of K.E of system of N - particles.
Then the K.E of the system
T =
1
2
m

r
2
C
+
1
2
N

i=1
m
i

2
i
=
1
2
m

r
2
C
+
1
2
_
V

2
i
dV

is the position vector of volume element dV relative to mass


centre c.m.
m is the mass of the body. The rst term is called translational
K.E. and second term is called rotational K.E.
Let assume that the body be rotating with angular velocity

.
To nd

2
= (

)
2
=

= (

__

2
=

[(

]
25
=


_

2
(

2
=


_

2
(

Let = x

i + y

j + z

k =
x

i +
y

j +
z

k

_

2
( )

_
_
x
2
+ y
2
+ z
2
_

i (x
x
+ y
y
+ z
z
)

x
i
_

__
y
2
+ z
2
_

x
xy
y
xz
z

=
_
__
y
2
+ z
2
_

x
(xy
y
+ xz
z
)

i
+ [ ]
j
+ [ ]
k
_

T =
1
2
_
V

2
dV
=
1
2

_
_
_
_
V

___
y
2
+ z
2
_

x
(xy
y
+ xz
z
)

i
+[ ]
j
+ [ ]
k
_

_
dV
Dene m.I and product of inertial as follows. Let
I
xx
=
_
V
_
y
2
+ z
2
_

dV
I
yy
=
_
V
_
z
2
+ x
2
_

dV
I
zz
=
_
V
_
x
2
+ x
2
_

dV
I
xy
= I
yx
=
_
V
(xy)

dV
I
yz
= I
zy
=
_
V
(yz)

dV
26
I
zx
= I
xz
=
_
V
(zx)

dV
T
rot
=
1
2
I
xx

2
x
+
1
2
I
yy

2
y
+
1
2
I
zz

2
z
+ I
xy

y
+ I
yy

z
+ I
zx

x
=
3

i=1
3

j=1
1
2
I
ij

j
=
1
2

T
I

=
1
2
_
_
_
_

z
_
_
_
_
T
_
_
_
_
I
xx
I
xy
I
xz
I
yx
I
yy
I
yz
I
zx
I
zy
I
zz
_
_
_
_
_
_
_
_

z
_
_
_
_
=
1
2
(

)
T

I (

)
Kinetic Enery of rigid body =
1
2
m

r
c
+

T
I

Angular Momentum of a system :


The angular momentum of a system of N particles and the total
mass M about a xed point O is equal to the angular momentum
about of a single particle of mass m which is moving with centre of
mass plus the angular momentum of the system about the centre of
mass m.
Find the angular momentum of a system of particles about a xed
point .
moment of momentum = Angular momentum.
Proof :
Let us consider a system of N - particles Let m
i
is the mass of the
i
th
particle whole position vectors given by

r
i
.
The angular momentum of the i
th
particle = r
= r mv
27
= m

r r
AM =

r
i
m

r
Angular momentum of the system H =
N

i=1

r
i
m

r .
But
N

i=1
m
i

i
= 0
N

i=1
m

i
= 0
Angular momentum of the system
=
_

r
C
_
m + 0 + 0 +
N

i=1

i
m

r
C
m

r
C
+
N

i=1

i
m

H =
_

r
C
m

r
C
_
+ H
C
wher H
C
is the angular momentum of the system relative to a
centre of mass.
Note :
T
ra
=
1
2

) dV
=
1
2

H
C
Angular momenum of a rigid body :
Le dV be small volume element of a rigid body having density .
Each element of the rigid body is having translation and relation.
We can assume that the rigid body is having instantaneous axis of
rotation.
The typical volume element dV can be chosen so small.
28
Its rotational K.E is negligible compared with its translational K.E
each element of rigid body can be considered as particle of innite
decimal mass.
The angular momentum of a systme of particle

H =

r
C
m

r
C
+
N

i=1

i
m

C
.
In the case of rigid body

H =

r
C
m

r
C
+
_
V

_
dV
But

, where

is the angular velocity of the rigid body.

) =

2
(

Let us consider a cartesian system with its origin at the centre of mass.

i
= x

i + y

j + z

k ,

=
x

i +
y

j +
z

2
( ) =

_
x
2
+ y
2
+ z
2
_

x
(x
x
+ y
y
+ z
z
)

x
I
=

_
y
2
+ z
2
_

x
(xy
y
+ xz
z
)

i
=
_

_
y
2
+ z
2
_

x
(xy
y
+ xz
z
)
_
i
I
xx
=
_
V
_
y
2
+ z
2
_
dV
I
yy
=
_
V
_
z
2
+ x
2
_
dV
I
zz
=
_
V
_
x
2
+ y
2
_
dV
I
xy
= I
yx
=
_
V
(xy) dV
I
yz
= I
zy
=
_
V
(yz) dV
I
zx
= I
xz
=
_
V
(zx) dV
29

_
V

_
dV
=
_
V

2
(

dV

=
_
V
_
__
y
2
+ z
2
_

x
xy
y
xz
z

i
+
_

_
z
2
+ x
2
_

y
yz
x
yx
x

j
+
_

_
x
2
+ y
2
_

x
zx
y
zy
y

k
_
dV
= I
xx

i + I
yy

j + I
zz

k + I
xy

i
+I
yz

j + I
zx

k + I
xz

i + I
yx

j + I
zy

H
C
= (I
xx

x
+ I
xz

z
+ I
xy

y
)

i + (I
yy

y
+ I
yz

z
+ I
yx

x
)

j
+(I
zz

z
+ I
zx

x
+ I
zy

y
)

H
C
= I

where I =
_
_
_
_
I
xx
I
xy
I
xz
I
yx
I
yy
I
yz
I
zx
I
zy
I
zz
_
_
_
_

H =

r
C
m

r
C
+

H
C
where

H
C
= I

.
Problem:
Prove that T
rot
=
1
2

H
C
.
Proof:
The rotational K.E of a rigid body M given by
T =
1
2
_
V

dV (1)
is the density of the rigid body,

is the position of the rigid
body is the position vector of elementary volume dv with respect to
centre of mass.
Let

be the angular velocity of the body. Let

r
C
be the position
30
vector of centre of mass w.r. to origin O. Then

H =
_

r
C
m

r
C
_
+

H
C
where

H
C
=
_
V
(

)) dV (2)
But

2
=

)
= (

=


_

_
=


_

__
T
rot
=
1
2
_
V

2
dV
=
1
2
_
V

)] dV
=
1
2

H
C
( using (2))
Angular momentum with respect to an arbitrary point :
Let P be arbitrary point,

r
P
is the position vector of P w.r. to O,
m
i
is the position of the i
th
particle with position vector

r
i
, c.m. is
the centre of mass of the system,

i
is the position vector of the i
th
particle w.r. to P.
Angular momentum of the i
th
particle w.r.to p =
i
m
i

i
.
Angular moment of the system about the position P is

H
P
=
N

i=1

i
m
i

i
=

r
i

r
P
31
But

r
P
=

r
C

i
=

r
i

_

r
C

C
_
=

r
i

r
C
+

i
=

r
i

r
C
+

H
P
=
N

i=1
_

r
i

r
C
+

C
_
m
i
_

r
i

r
C
+

C
_
=
N

i=1
m
i
_

r
i

r
i

r
i

r
C
+

r
i

r
C

r
i
+

r
C

r
C

r
C

C
+

r
i

r
C
+

C
_
AM of the system about the arbitrary point
=
N

i=1

r
i
m
i

r
i

N

i=1
m
i

r
i

r
C
+
N

i=1
m
i

r
i

.

r
C

N

i=1
m
i

r
i
+

r
C
m

r
C

r
C
m

C
+

i=1
m
i

C
m

r
C
+

C
m

C
But

r
C
=
N

i=1
m
i

r
i
N

i=1
m
i

i=1
m
i

r
C
=
N

i=1
m
i

r
i

r
C
=
N

i=1
m
i

r
i
32
AM of the system about the arbitrary point

H
P
=

H m

r
C

r
C
+ m

r
C

r
C
m

r
C
+

r
C
m

r
C

r
C
m

C
+

C
m

r
C

C
m

r
C
+

C
m

C
=

H

r
C
m

r
C
+

C
m

C
=
N

i=1

r
i
m
i

r
i

r
C
m

r
C
+

C
m

C
1. Considering the K.E of a system of N particles show with usual
notation that
T =
1
2
m

r
2
C
+
1
2
N

i=1
m
i

2
i
2. Prove with usual notation that
T
rot
=
1
2

H
C
3. Obtain the expression for the rational K.E. (Rigid body on sys-
tem of particle)
Generalized Momentum :
Consider a system with n - generalized co-ordinates q
1
, q
2
, , q
n
.
Let T be K.E of the system. V is the potential energy of the
system. Let us dened the Lagrangian function L
_
q,

q, t
_
as follows
L = T V .
The generalized momentum p
i
associated with q
i
is dened as
p
i
=
L

q
i
=
T

q
i
_

_
V

q
i
_
= 0
_
.
Note : i) L is almost a function of quadratic in q

s. Hence p
i
is a
linear function of q

s. If V = V (q, t), then


L
q
i
=
T
q
i

V
q
i
.
ii) Consider a particle of mass in with cortesian co-ordinates (x, y, z).
33
The K.E of the particle is given by
T =
1
2
m
_

x
2
+

y
2
+

z
2
_
p
x
=
T


x
=
1
2
m2

x = m

x
Similarly p
y
= m

y and
p
z
= m

z
p
x
is the x component of the linear momentum. If the position of the
particle is given by spherical polar co-ordinates (r, , ) ,then
T =
1
2
m
_

r
2
+ r
2

2
+ r
2

2
sin
2

_
p
r
=
T

r
=
1
2
m2

r = m

r
p

=
T

=
1
2
mr
2
2

= mr
2

=
T

=
1
2
mr
2
2

sin
2
= mr
2
sin
2

p
r
is called linerar momentum component radial direction.
p

is called horizandal component of hte angular momentum.


p

is called vertical component of angular momentum.


1. Three particles are conneeted by 2 rigid rods having a joint be-
tween them to form a system verticla force F and a momentum
M are applied as shown in gure. The conguration of the sys-
tem is given by the ordinary co-ordinates (x
1
, x
2
, x
3
) or by the
generalized co-ordinates (q
1
, q
2
, q
3
) where x
1
= q
1
+q
2
+
1
2
q
3
; x
2
=
q
1
q
3
; x
3
= q
1
q
2
+
1
2
q
3.
Find the expression for K.E and gen-
eralized momentum.
Solution :
Let T be the K.E. of the system.
T =
1
2
m
_

x
2
1
+

x
2
2
+

x
2
3
_
34
x
1
= q
1
+ q
2
+
1
2
q
3
x
2
= q
1
q
3
x
3
= q
1
q
2
+
1
2
q
3

x
1
=

q
1
+

q
2
+
1
2

q
3

x
2
=

q
1


q
2

x
3
=

q
1


q
2
+
1
2

q
3
T =
1
2
m
_
3

q
2
1
+ 2

q
2
2
+
3
2

q
2
3
_
p
1
=
T

q
1
=
1
2
m3
_
2

q
1
_
= 3m

q
1
p
2
=
T

q
2
=
1
2
m4

q
2
= 2m

q
2
p
3
=
T

q
3
=
1
2
m
3
2
2

q
3
=
3m
2

q
3
p
1
, p
2
, p
3
are called the generalized moment equal are inertial co-eceient.
2 Three particles are connected by two rigid rods having a joint
between them to form the system shown in the gure. A virtual
force F and momentum m are applied to the system as shown the
conguration of the system is given by the ordinary co-ordinates
(x
1
, x
2
, x
3
) or by the gneralized co-ordinates (q
1
, q
2
, q
3
), where
x
1
= q
1
+ q
2
+
1
2
q
3
; x
2
= q
1
q
3
; x
3
= q
1
q
2
+
1
2
q
3.
Find the
generalized force Q
1
, Q
2
, Q
3
l l
m
m
m
M
3
4
l
2
x
3
x
1
x
Solution :
Assume small motions the applied force on the systemF
1
=
3F
4
, F
2
=
F
4

m

, F
3
=
m

.
35
Let Q
1
, Q
2
, Q
3
be the corresponding generalized forces on the sys-
tem.
Q
i
=
3N

j=1
F
j
x
j
q
i

3N

j=1
F
j
x
j
q
i
Q
1
= F
1
x
1
q
1
+ F
2
x
2
q
1
+ F
2
x
3
q
1
=
3F
4
(1) +
_
F
4

m

_
+
m

=
3F
4
+
F
4
= F
Q
2
= F
1
x
1
q
2
+ F
2
x
2
q
2
+ F
2
x
3
q
2
=
3F
4
(1) +
_
F
4

m

_
(0) +
m

(1)
=
3F
4

m

Q
1
= F
1
x
1
q
1
+ F
2
x
2
q
1
+ F
2
x
3
q
1
=
3F
4
_
1
2
_
+
_
F
4

m

_
(1) +
m

_
1
2
_
=
3F
8

F
4
+
m

+
m
2
=
F
8
+
3m
2
Thus Q
1
= F, Q
2
=
3F
4

m

and Q
3
=
F
8
+
3m
2
.
Energy of momentum conservative systems :
Let (x, y, z) be the position of a particle. Let

F be the total force
acting on the partical with components
F
x
=
V
x
, F
y
=
V
y
, F
z
=
V

where V (x, y, z) is the potential energy function depending on position


only. In this case F is called conservative force.
Book work : Prove that for any constervative force
_

F d

r = 0
Proof : Let

F be the conservative force acting on the particele at
36
(x, y, z). Let V (x, y, z) be the potential energy of the particle.
F
x
=
V
x
, F
y
=
V
y
, F
z
=
V
y
Workdone =
_
F
x

i + F
y

j + F
z

k
_
d
_
x

i + y

j + z

k
_
W =
_
Fdr
dW =

F dr = F
x
dx + F
y
dy + F
z
dz
=
_

V
x
_
dx +
_

V
y
_
dy +
_

V
z
_
dz
=
_
V
x
dx +
V
y
dy +
V
z
dz
_
dW = dV (x, y, z)
Thus dW is an exact dierential equation

F d

r = dV
Let W be the workdone by the force

F is the moving particle from


A to B.
Total workdone = W =
B
_
A
dV =
B
_
A
dV = [V ]
B
A
= V
A
V
B
Workdone on the particle is depending on the initial and nal
positions.
Workdone is moving around a closed curve is
_

F d

r = 0
_

A =

B
_
37
UNIT II
Lagranges Equation :
Let us consider a system of N particles whose positions relative to
an inertial frame are given by the cartesian co-ordinates x
1
, x
2
, , x
3N
.
Kinetic energy of the system
T =
1
2
3N

k=1
m
k

x
2
k
(1)
where m
1
= m
2
= m
3
, m
4
= m
5
= m
6
is the mass of the rst particle,
second particle respectively.
Let q
1
, q
2
, , q
n
be the generalized co-ordinates of the system.x
k
=
x
k
(q, t) , k = 1, 2, 3, , 3N where we assume that tehse functions are
twice dierentiable with respect to the q

s and t. We nd that

x
k
_
q,

q, t
_
=
n

i=1
x
k
q
i

q
i
+
x
k
t

x
k
=
x
k
q
1

q
1
+
x
k
q
2

q
2
+ +
x
k
q
n

q
n
+
x
k
t

x
k
=
n

i=1
x
k
q
i

q
i
+
x
k
t
Put in (1), we get
T =
1
2
3N

k=1
m
k
_
n

i=1
x
k
q
i

q
i
+
x
k
t
_
2
=
1
2
3N

k=1
m
k
_
n

i=1
n

j=1
x
k
q
i
x
k
q
j

q
i

q
j
+ 2
n

i=1
x
k
q
i

q
i
x
k
t
+
_
x
k
t
_
2
_
=
1
2
n

i=1
n

j=1
_
3N

k=1
m
k
x
k
q
i
x
k
q
j
_

q
i

q
j
+
n

i=1
_
3N

k=1
m
k
x
k
q
i
x
k
t
_

q
i
+
1
2
3N

k=1
m
k
_
x
k
t
_
2
.
38
Let
3N

k=1
m
k
x
k
q
i
x
k
q
j
= m
ij
and
3N

k=1
m
k
x
k
q
i
x
k
t
= a
i
T =
1
2
n

i=1
n

j=1
m
ij

q
i

q
j
+
n

i=1
a
i

q
i
+
1
2
3N

k=1
m
k
_
x
k
t
_
2
T
_
q,

q, t
_
= T
2
+ T
1
+ T
0
.
where T
2
is a homogeneous quadratic in q

s, T
1
is homogeneous linear
function in

q
i
, T
0
is a function of q

s and t

s in general.
Note:
(i) T is a functin of
_
q,

q, t
_
.
T = T
_
q,

q, t
_
(ii) For a sceleronomic system T
0
= 0
T
1
= 0
since
x
k
t
= 0
In this case T = T
2
.
2. Obtain Lagranges equatin from D Alemberts principle interms
of generalized co-ordinates.
Proof :
Let us assume that the system has N particles with 3N cartesian
co-ordinates from D alemberts principle, we have
3N

k=1
_
F
k
m
k

x
k
_
x
k
= 0 (1)
where F
k
is the applied force component associated with x
k
.
39
Let us assume that the system is determined by n-generalized co-
ordinates q
1
, q
2
, , q
n
.
x
k
= x
k
(q, t)
x
k
= x
k
(q
1
, q
2
, , q
n
, t)
x
k
=
n

i=1
x
k
q
i
q
i
+
x
k
t
t
(1)
3N

k=1
_
F
k
m
k

x
k
_
n

i=1
x
k
q
i
q
i
= 0
n

i=1
3N

k=1
_
F
k
m
k

x
k
_
x
k
q
i
q
i
= 0 (2)

x
k
=
n

i=1
_
x
k
q
i

q
i
_
Dierentiating w.r.to

q
i


x
k

q
i
=
x
k
q
i
(3)
d
dt
_


x
k

q
i
_
=
d
dt
_
x
k
q
i
_
=


x
k
q
i
T =
1
2
3N

k=1
m
k

x
2
k
p
i
=
T

q
i
=
1
2
3N

k=1
m
k
2

x
k


x
k
q
i
=
3N

k=1
m
k

x
k


x
k
q
i
d
dt
_
T

q
i
_
=
3N

k=1
_
m
k

x
k


x
k
q
i
+ m
k

x
k
d
dt
_


x
k
q
i
__
40
=
3N

k=1
m
k
x
k
q
i

x
k
+
3N

k=1
m
k

x
k


x
k
q
i
(4)
T
q
i
=
1
2
3N

k=1
m
k
2

x
k


x
k
q
i
(5)
d
dt
_
T
q
i
_

T
q
i
=
3N

k=1
m
k

x
k
x
k
q
i
(6)
Let us dene the generalized force Q
i
asq
Q
i
=
3N

k=1
F
k
x
k
q
i
By (2) we have
n

i=1
_
3N

k=1
F
k
x
k
q
i

3N

k=1
m
k

x
k
x
k
q
i
_
q
i
= 0
n

i=1
_
Q
i

d
dt
_
T

q
i
_
+
T
q
i
_
q
i
= 0 (7)
This is the Lagranges form of D Alemberts principle interms of gen-
eralized co-ordinates.
The above discussion the system is under instantaneous constraints.
Let us assume the system be holonomic and described by n- inde-
pendet generalized co-ordinates q
1
, q
2
, , q
n
.
The q

i
s are independent.
Hence the co-eceient of qs in equation (7) must be zero. qs are
independent.
co-ecient of q = 0
d
dt
_
T

q
i
_

T
q
i
= Q
i
, i = 1, 2, 3, , n.
These equations are Lagrange equations from holonomic system.
Book work : Obtain the standard form of Lagranges equation for a
41
holonomic system.
Proof : Let us assume that all the generalized forces are derivable form
a potential function V (q, t) as let the system is holonomic conservative.
Q
i
=
dV
dq
i
=
d
dt
_
T

q
i
_

T
q
i
=
V
q
i
d
dt
_
T

q
i
_

T
q
i
+
V
q
i
= 0
d
dt
_
T

q
i
_


q
i
(T V ) = 0
Let L = T V (Lagarangian function) T = L + V.
T

q
i
=
L

q
i
_
L
_
q,

q, t
__
d
dt
_
T

q
i
_

q
i
= 0, i = 1, 2, 3, , N.
Note :
1. The standard form of Lagranges equations
d
dt
_
T

q
i
_

L
q
i
= 0, i = 1, 2, 3, , N.
since p
i
=
L

q
i

d
dt
(P
i
)
L
q
i
= 0
where p
i
=
L
q
i
, i = 1, 2, 3, , n.
2. Let us assume the generalized forces Q
i
is not completly deriva-
42
tive from a potential function
Q
i
=
V
q
i
+ Q

i
we have
d
dt
_
T

q
i
_

T
q
i
= Q
i
d
dt
_
T

q
i
_

T
q
i
=
V

q
i
+ Q

i
d
dt
_
T

q
i
_

q
i
+
T
q
i
= Q

i
Let L = T V.
d
dt
_
T
q
i
_

q
i
= Q

i
, i = 1, 2, 3, , n.
Book work : Obtain the Lagranges equation for a non-holonomic
system.
Proof : Let us assume that system has m non holonomic constraint
equation
n

i=1
a
ji
dq
i
+ a
ji
dt = 0 (j = 1, 2, 3, , m) .
Let us assume a virtual displacement consistent with the con-
straints.
Now we have
n

i=1
a
ij
q
i
= 0, j = 1, 2, 3, , m (1)
The constraints are assumed to be workless. Hence we have,
n

i=1
c
i
q
i
= 0 (2)
43
where c
i
is the generalized constraints force
Multiply equation (1) by Lagranges multiply
j

j
n

i=1
a
ji
q
i
= 0, j = 1, 2, 3, , m (3)
Adding these m equations and subtracting from (2) we get
(2) (3)
n

i=1
_
c
i

j=1

j
a
ji
_
q
i
= 0
Choose the

s arbitrary such that


c
i
=
m

j=1

j
a
ji
, i = 1, 2, 3, , m
of the generalized forces are not wholly derivable form a potential
function.
QQ
i
=
V
q
i
+ Q

i
;
we have
d
dt
_
L

i
_

i
= Q

i
, i = 1, 2, 3, , n
In our discussion Q

i
nothing but the c
i
d
dt
_
L

q
i
_

L
q
i
=
m

j=1

j
a
ji
, i = 1, 2, 3, , n.
This is the standard form of Lagranges equation for the non-holonomic
systyem.
1. Find the dierential equation of motion for a spherical pendulum
of length (using Lagranges equation).
Solution : Let use the spherical co-ordinates and . is mea-
sured from the upward vertical, is the angle between a vertical plane
through supporting point O and a vertical plane containing the pen-
44
dulum.
The spherical co-ordinates are (r, , ) for this problem.
r = (constant) r

= 0
Let T be the K.E. and V is the P.E.
T =
1
2
m
_

2
+
2

2
sin
2

_
V = mg ( cos ) = mg cos
L = T V
=
1
2
m
_

2
+
2

2
sin
2

_
mg cos
The Lagranges equation is given by
d
dt
_
L

q
i
_

L
q
i
= 0. For our
problem, q
i
= , q
2
= 0, q
3
= , we have
d
dt
_
L

= 0
d
dt
_
1
2
m
2
2

_
1
2
m
2

2
2 sin cos + mg sin
_
= 0
m
2

m
2

2
sin cos mg sin = 0
Dividing by m

2
sin cos g sin = 0 (1)
q
f
mg
sin l q
cos l q
l
O
180 q -
45
Also
d
dt
_
L

= 0
d
dt
_
1
2
m
2
2

sin
2

_
0 = 0
d
dt
_
m
2

sin
2

_
= 0
Dierentiating w.r.to t
m
2
sin
2

+ 2m

2
sin cos

= 0
Dividing by m
cos h l q =
cos
h
l
q =
g
h
m
sin
2

+ 2 sin cos

= 0 (2)
sin

+ 2 cos

= 0
we have
L

= 0
d
dt
_
L

_
= 0
46

= p

= constant
i.e., m
2
sin
2
.

= p

= constant
where p

is the angular momentum above a vertical axis ow the sup-


porting point.
Inertial Co-eceient :
The co-ecient of the q is the expression for the generalized mo-
ments are known an inertial co-eceient.
In the Book work 2, 3n, 2m,
3m
2
are inertial co-eceient.
2 A doubel pendulum consists of particles supported by mass less
rods assuming that all motions take place in a vertical plane.
Find the dierential equation of the motion linearise these equa-
tions assuming small motions.
Solution :
Let T be the K.E of the system and V is the potential energy of
the system.
Let L = T V. Let V be the velocity of the particle.
V = The velocity of upper particle + velocity of the lower particle.
l
f
&
l
f
&
l
q
&
l
f q -
f q -
q
O
q
m
B
T =
1
2
mv
2
1
+
1
2
mv
2
2
v
1
=

v
2
=
_

2
+
2

2
+ 2

cos ( )
T =
1
2
m
2

2
+
1
2
m
_

2
+
2

2
+ 2

cos
_
47
=
1
2
m
2
_
2

2
+

2
+ 2

cos ( )
_
V = mg [ cos + cos + cos ]
= mg (2 cos + cos )
L = T V
=
1
2
m
2
_
2

2
+

2
+ 2

cos ( )
_
+ mg (2 cos + cos )
l cosq l
B
A
C
std level
cosf l
cosq l
( )
1
p mg
f
f
mg
The equation
d
dt
_
L

= 0
d
dt
_
1
2
m
2
2.2

+
1
2
m
2
.2

cos ( )
_

1
2
m
2
2

{sin ( ) (1)} + (2mg sin ) = 0


48
l
O
q
f
g
l
m
m
2
d
dt
_
2

cos ( )
_
m
2

sin ( ) + 2mg sin = 0


m
2
_
2

cos ( ) +

(sin ( ))
_

_
m
2

sin ( ) + 2mg sin


_
= 0
m
2
_
2

cos ( )

2
sin ( )
_
+ 2mg sin = 0 (1)
Now the equation
d
dt
_
L

= 0
d
dt
_
1
2
m
2
_
2

+ 2

cos ( )
_

1
2
m
2
2

(sin ( )) mg sin
_
= 0
m
2
d
dt
_

+

cos ( )
_
+ m
2

sin
_

_
+ mg sin = 0
m
2
_

+

cos ( ) +

2
sin ( )
_
+ mg sin = 0 (2)
49
(1) and (2) are the dierential equation of motion linearise the
equation. Let us assume the system having small oscillations (, )
are small.
sin = , sin =
cos ( ) = 1, sin ( ) =
Neglecting higher order terms equation (1) becomes
m
2
_
2

_
+ 2mg = 0
and equation (2) becomes
m
2
_

_
+ mg = 0
3 A block of mass m
2
can slide on another block of mass m
1
which
in turn, slides on a horizontal surface, as shown in the gure.
Using x
1
and x
2
as co-ordinates, obtain the dierential equations
of motion. Solve for the accelerations of the two blocks as they
move under the inuence of gravity, assuming that all surfaces
are frictionless. Find the fore of interaction between the blocks.
Solution : Let x
1
be the displacement of m
1
, let x
2
be the dis-
placement of m
2
relative to m
1
. Let v
1
be the velocity of m
1
and v
2
be the velocity of m
2
relative to m
1
.
T =
1
2
m
1
v
2
1
+
1
2
m
2
v
2
2
v
1
= x
1
v
2
=
_
x
2
1
+ x
2
2
2 x
1
x
2
cos 45
0
=
_
x
2
1
+ x
2
2

2 x
1
x
2
T =
1
2
m
1
x
2
1
+
1
2
m
2
_
x
1
+ x
2

2 x
1
x
2
_
.
50
2
m
2
x
1
x
0
45
Let V be the P.E. of the system
V m
2
gx
2
sin 45
0
=
1

2
m
2
gx
2
L = T V
=
1
2
m
1
x
2
1
+
1
2
m
2
_
x
2
1
+ x
2
2

2 x
1
x
2
_
+
1

2
m
2
gx
2
x
1
Lagranges equation is
d
dt
_
L
x
1
_

_
L
x
1
_
= 0
d
dt
_
1
2
m
1
2 x
1
+
1
2
m
2
2 x
1
_

2
m
2
x
2
= 0
m
1
x
1
+ m
2
x
1

2
m
2
x
2
= 0
(m
1
+ m
2
) x
1

2
m
2
x
2
= 0 (1)
[width=6cm]m12
x
2
Lagranges equation is
d
dt
_
L
x
2
_

_
L
x
2
_
= 0
d
dt
_
1
2
m
1
2 x
2

2
m
2
x
1
_

2
m
2
g = 0
m
2
x
2

2
m
2
x
1
=
1

2
m
2
g (2)
51
(1) + (2)
1

2
m
2
x
2

2
m
2
x
1
=
1

2
m
2
g
(1) (m
1
+ m
2
) x
1
=
1

2
_
1

2
m
2
x
1
+ m
2
g
_
_
m
1
+ m
2

m
2
2
_
x
1
=
1

2
m
2
g

2
x
1
=
m
2
g
2m
1
+ m
2
(1) (m
1
+ m
2
)
m
2
g
2m
1
+ m
2
=
1

2
m
2
x
2
x
2
=

2
(m
1
+ m
2
) g
2m
1
+ m
2
4 A particle of mass m can side without friction on the inside of a
small tube which is bent in the form of a circle of radius r, the
tube rotates about a vertical diameter with a constant angular
velocity , as shown in gure write the dierential equation of
motion.
Solution :
Let O be the centre of the tube and r be the radius of the tube.
Let T be the K.E of the particle of mass m,V is the P.E. of the
particle.
T =
1
2
m
_
r
2

2
+ r
2

2
sin
2

_
=
1
2
mr
2
_

2
+
2
sin
2

_
V = mgh = mgr cos
L = T V
=
1
2
mr
2
_

2
+
2
sin
2

_
mgr cos , r is constant
52
The Lagranges equation is
d
dt
_
L

= 0
d
dt
_
1
2
mr
2
2

_
1
2
mr
2

2
2 sin cos + mgr sin
_
= 0
d
dt
_
mr
2

_
1
2
mr
2

2
2 sin cos + mgr sin
_
= 0
mr
2

mr
2
sin cos mgr sin = 0
5 Mass spring system : A particle of mass m is connected by a
massless spring of stiness K and unstressed length r
0
to a point
P which is moving along a circubr path of radius a at a uniform
angular rate . Assuming that the particle moves without friction
on a horizontal plane, nd the dierential equations of motion.
Solution :
Resolving r and r

along a and r

.
In this probem, we have dealing with a single particle of mass m.
Let O taken as origin of refeence length .
r
P
is the position vector of the point P w.r.to the point O.

is the velocity of the particle w.r.to P.


Let T be the K.E. of the particle, V is the P.E of th+e particle.
T =
1
2
m

r
P
+
1
2
m

2
+

r
P
m

O
is the position vector of the system w.r.to .
In the above case,
C
=
T =
1
2
m

r
P
+
1
2
m

2
+

r
P
m

r
P
= a

2
= r
2
+ r
2

r
P
m

= ma
_
r sin ( t) + r

cos ( t)
_
53
T =
1
2
ma
2

2
+
1
2
m
_
r
2
+ r
2

2
_
+ma
_
r sin ( t) + r

cos ( t)
_
V =
1
2
K (r r
0
)
2
L = T V
=
1
2
ma
2

2
+
1
2
m
_
r
2
+ r
2

2
_
+ma
_
r sin ( t) + r

cos ( t)
_

1
2
K (r r
0
)
2
The Lagranges r equation is
d
dt
_
L
r
_

L
r
= 0
d
dt
_
1
2
m2 r + ma sin ( t)
_

_
1
2
m2r

2
+ ma
_

cos ( t)
_

1
2
K2 (r r
0
)
_
= 0
m r + ma cos ( t)
_


_
mr

ma

cos ( t) + K (r r
0
) = 0
m r ma
2
cos ( t) mr

2
+ K (r r
0
) = 0
The Lagranges equation is given by
d
dt
_
L

= 0
d
dt
_
1
2
m2r
2

+ ma cos ( t)
_

ma r cos ( t)
_
mar

sin ( t)
_
= 0
mr
2

+ 2m r

+ ma r cos ( t)
mar

sin ( t) + mar
2
sin ( t)
ma r cos ( t) + ma r sin ( t) = 0
m r + 2m r

+ ma
2
r cos ( t) = 0
54
Knife edge problem :
Example of a non-holonomic rheonomic system.
Two particles are connected by a rigid massless rod of length
which rotates in a horizontal plane with a constant angular vlocity
. Knife - edge supports at the two particles prevent either particle
from having a velocity component along the rod, but the particles can
slide without friction in a direction perpendicular to the rod. Find the
dierential equations of motion. Solve for x, y and the constraint force
as functions of time if the center of mass is initially at the origin and
has a velocity v
0
in the positive y direction.
Solution :
Let (x, y) be the co-ordinates of the CM at time the system has
translational and rotational K.E
Velocity component along the rod = 0
( x
1
+ x
2
) cos t + ( y
1
+ y
2
) sin t = 0 (1)
T =
1
2
m
_
x
2
1
+ y
2
1
_
+
1
2
m
_
x
2
2
+ y
2
2
_
=
1
2
m
_
x
2
1
+ x
2
2
+ y
2
1
+ y
2
2
_
=
1
2
m
_
2 x
2
+ 2 y
2
+
2

2
2
_
= m
_
x
2
+ y
2
_
+ +m
2

2
4
V = 0
Lagranges equation of for non-haolonomic system is
d
dt
_
L
q
i
_

L
q
i
=
m

i=1

j
a
ij
=
1
a
11
.
The velocity of centre of mass exsited only in a direction perpen-
dicular to the rod. Hence it has no velocity along the rod.
55
x
1
= x

2
cos t
y
1
= y

2
sin t
x
2
= x +

2
cos t
y
2
= y +

2
sin t
x
1
= x +

2
sin t
y
1
= y

2
cos t
_

_
(2)
x
2
= x

2
sin t
y
2
= y +

2
cos t
From (1) and (2)
x cos t + y sin t = 0
x cos t + y cos
_
90
0
t
_
= 0
x cos t + y sin t = 0 (A)
_
x +

2
sin t + x

2
sin t
_
cos t +
sin t
_
y

2
cos t + y +

2
cos t
_
= 0
2 x cos t + 2 y sin t = 0
cos tdx + sin tdy = 0
n

i=1
a
ji
dq
i
+ a
jt
dt = 0
a
11
x + a
12
y = 0, q
1
= x
1
, q
2
= y
2
56
a
11
= cos t
a
12
= sin t
_
n

i=1
a
ji
a
ij
= 1
d
dt
_
L
x
_

L
x
= a
11
L = T V
= m
_
x
2
+ y
2
_
+
m
4

2

2
L
x
= 2m x

d
dt
(2m x) 0 =
n

i=1

j
a
ji
=
1
cos t
2m x =
1
cos t (B)
d
dt
_
L
y
_

L
y
= a
12
d
dt
(2m y) =
1
a
12
2m y =
1
sin t (C)
(C)
(B)
tan t =
y
x
xcos t + y sin t = 0

x
y
= tan t
y
x
=
x
y
2 x x + 2 y y = 0
2 xd ( x) + 2 yd ( y) = 0
d
dt
_
x
2
+ y
2
_
= 0
d
dt
_
2 x
2
+ 2 y
2
_
= 0
2 x
2
+ 2 y
2
= constant
57
Initially x = 0, y = 0,
0 + v
2
0
= 0,
x
2
+ y
2
= v
2
0
xcos t + y sin t = 0
Hence the centre of mass moves with constant velocity, satisfying those
equation, we have
x = v
0
sin t
y = v
0
cos t
dx
dt
= v
0
sin t
dx = v
0
sin tdt
_
dx =
_
v
0
sin tdt
x =
v
0

cos t + C
1
x = 0, t = 0
0 =
v
0

+ C
1
C
1
=
v
0

x =
v
0

cos t
v
0

=
v
0

(cos t 1)
y
0
= 0; t = 0
C
2
= 0
y =
v
0

sin t
Eliminating t by , x and y we have circular path. The generalized
58
forces are
C
i
=
m

i=1

j
a
ji
, j = 1, 2, 3, , m
C
1
=
1
a
11
C
1
= cos t
2m x
cos t
_

1
=
2m x
cos t
_
x = v
0
sin t
x = v
0
cos t
C
1
= 2m(v
0
cos t)
C
2
=
2
a
12
=
2m y
cos t
sin t
=
2mv
0
sin t cos t
cos t
= 2mv
0
sin t
Note:
In the above case,
x =
v
0

(cos t 1)
x +
v
0

=
v
0

cos t
_
x +
v
0

_
2
= y +
v
2
0

2
Hence the system moves in a circular path with radius v
0
/.
2.3 Integrals of the motion:
Ignorable co-ordinates:
Let the holnomic system be described by n standard Lagranges
equation
d
dt
_
L
q
i
_

L
q
i
= 0, i = 1, 2, 3, , n
Let (q, q, t) containing all q
1
, q
2
, , q
n
, but some of the q

s, say q
1
, q
2
, , q
n
are missing from the Lagrangian. These k co-ordinaties are called ig-
norable co-ordinates. Since
L
q
i
is zero for each ignorable co-ordinate,
59
it follows that
d
dt
_
L
q
i
_
= 0, (i = 1, 2, 3, , k) ,
or
p
i
=
L
q
i
=
i
, i = 1, 2, 3, , k
where the

s are constants evaluated from the initial conditions. Hence


we nd that the generalized momentum corresponding to each ignor-
able co-ordinate is constant, that is, it is an integral of the motion.
Example :
Let us consider the Keplers problem, that is, the problem of the
motion of a particle of unit mass which is attracted by an inverse square
gravitational force to a xed point O. Using polar co-ordinates, the
kinetic and potential evergies are
T =
1
2
_
r
2
+ r
2

2
_
V =

r
where is a positive constant known as the gravitational coeceient.
The Lagrangian function is
L = T V =
1
2
_
r
2
+ r
2

2
_
+

r
L is a independent co-ordinates
Lagrangian r equation is
d
dt
_
L
r
_

L
r
= 0
d
dt
( r)
1
2
_
2r

2
_
+

r
2
= 0
r r

2
+

r
2
= 0. (1)
Since does not appear explicity in the Lagrangian function, it is an
60
ignorable co-ordinate. The equation of motion is
d
dt
_
r
2

_
= 0
or
L

= constant
1
2
r
2
2

=

=

r
2
r
2
=

(2)
where is constant and is equal to the angular momentum of the
particle about the attracting centre O.
r r

2
r
4
+

r
2
= 0 ( using (2) in (1))
r

r
3
+

r
2
= 0
The Routhian function :
Consider a standard holonmic system with q
1
, q
2
, ., q
k
are ignor-
able co-ordinates
L = L(q
k+1
, , q
n
, q
1
, q
2
, , q
n
, t) .
Now let us dene a Routhian function
R(q
k+1
, , q
n
, q
k+1
, , q
n
,
1
,
2
, ,
k
, t)
as follows,
R = L
k

i=1

i
q
i
where
i
=
L
q
i
, i = 1, 2, 3, , k.
Obtain the Lagranges equation using Routhian funcion.
Solution :
61
Consider the holonomic system with n independent generalized
co-ordinates
q
1
, q
2
, .., .q
n
. Letq
1
, q
2
, .., q
k
be the ignorable co-ordinates. The
Legrangian function is
L = L(q
k+1
, , q
n
, q
1
, q
2
, , q
n
, t) .
Now
L
q
i
=
i
, i = 1, 2, 3, , k (1)
Now the Routhian funciton is given by
R = L
k

i=1

i
q
i
(2)
Subsitute the values for q
1
, q
2
, , q
k
Using (1) we have
R = R(q
k+1
, , q
n
, q
k+1
, , q
n
,
1
,
2
, ,
k
, t) .
Dierentiating (2)
R = L
k

i=1

i
q
i
(3)
L =
n

i=k+1
L
q
i
q
i
+
k

i=1
L
q
i
q
i
+
n

i=k+1
L
q
i
q
i
+
L
t
t
R =
n

i=k+1
R
q
i
q
i
+
n

i=k+1
R
q
i
q
i
+
n

i=1
L

i
+
R
t
t (4)

i=1

i
q
i
=
k

i=1
L
q
i
q
i
+
k

i=1
q
i

i
62
(3)

_
L
k

i=1

i
q
i
_
=
n

i=k+1
L
q
i
q
i
+
n

i=k+1
L
q
i
q
i

i=1
q
i

i
+
L
t
t (5)
From (4) and (5)
The corresponding coecients must be equal.
L
q
i
=
R
q
i
, i = k + 1, k + 2, k + 3, , n
L
q
i
=
R
q
i
, i = k + 1, k + 2, k + 3, , n
_

_
(6)
and
q
i
=
R

i
, i = 1, 2, 3, , k
L
t
=
R
t
.
Now let us substitute from equation (6) in to Lagrangian equations
and obtain
d
dt
_
R
q
i
_

R
q
i
= 0, i = k + 1, k + 2, k + 3, , n
R

i
= q
i
, i = 1, 2, 3, , k
Illustrate Keplers problem using Routhian method.
Solution :
Let us assume that the particle has unit mass. Let (r, ) be the
polar co-ordinates of the particle.
Let T be the kinetic energy of the particle. V is the potential evegy
of the particle.
T =
1
2
_
r
2
+ r
2

2
_
V =

r
L = T V
63
=
1
2
_
r
2
+ r
2

2
_
+

r
L

= 0
is ignorable solution
L

= r
2

=
Now the Routhian function is given by
R = L

=
1
2
_
r
2
+ r
2

2
_
+

r

=
1
2
r
2
+
1
2
r
2

2
r
4
+

r


2
r
2
=
1
2
r
2
+

r

1
2

2
r
2
Routhian equation is given by
d
dt
_
R
q
i
_

_
R
q
i
_
= 0
d
dt
_
R
r
_

_
R
r
_
= 0 (1)
Here there are two co-ordinates (r, )
L

= ( constant)
The Routhian equation is only at r
d
dt
( r)
_
_

r
2
_

1
2

2
_
2r
3
_
_
= 0
r +

r


2
r
3
= 0
or
r

2
r
3
+

r
= 0
64
Conservative system :
Write down the 3 condition dening a conservative system
and show that these conditions are suceient to ensure the
existence of an energy integral or Jacobi integral.
Proof :
1. The standard form of Lagranges equation (holonoic or nonholo-
nomic) applies.
2. The Legrangian function L is not an explicit function of time.
3. Any constraint equatins can be expressed in the dierential form
n

i=1
a
ji
dq
i
= 0, j = 1, 2, 3, , m
ie, all the coecients are equal to zero.
Consider the standard nonholonomic form of Lagranges equation
d
dt
_
L
q
i
_

L
q
i
=
n

i=1

j
a
ji
, i = 1, 2, , n (1)
where L(q, q) is not an explicit function of time.
Now let us consider the total derivative

jt
=

t
dL
dt
=
n

i=1
L
q
i
q +
n

i=1
L
q
i
q
i
(2)
(1)
L
q
i
=
d
dt
_
L
q
i
_

j=1

j
a
ji
(2)
dL
dt
=
n

i=1
_
d
dt
_
L
q
i
_

j=1

j
a
ji
_
q
i
+
n

i=1
L
q
i
q
i
65
=
n

i=1
_
d
dt
_
L
q
i
_
q
i
+
L
q
i
q
i
_

i=1
m

j=1

j
a
ji
q
i
=
d
dt
_
n

i=1
_
L
q
i
_
q
i
_

j=1
_
n

i=1
a
ji
q
i
_

j
dL
dt
=
d
dt
_
n

i=1
_
L
q
i
_
q
i
_
0
d
dt
_
L
n

i=1
_
L
q
i
_
q
i
_
= 0
d
dt
_
n

i=1
_
L
q
i
_
q
i
L
_
= 0
Integrating w.r.to t
n

i=1
_
L
q
i
_
q
i
L = h( constant)
h =
n

i=1
_
L
q
i
_
q
i
L
L =
n

i=1
_
L
q
i
_
q
i
h
h = T
2
T
0
+ V
=
n

i=1
_
L
q
i
_
q
i
L
This integral is called Jacobi integral or energy integral. This en-
ergy integral exists for all conservative system.
Book work :
Dene a natural system. Prove that for a conservative
system, Jacobi integral is not equal to the total evergy.
Proof :
For every conservative system
h = T
2
T
0
+ V
66
For the conservative system energy integral
h =
n

i=1
_
L
q
i
_
q
i
L (1)
L = T V
= T
2
+ T
1
+ T
0
V
We know that
T
2
=
1
2
n

i=1
m

j=1
m
ji
q
i
q
j
T
1
=
n

i=1
a
i
q
i
T
0
=
1
2
3N

k=1
m
k
_
x
k
t
_
2

i=1
L
q
i
q
i
=
n

i=1
T
q
i
q
i
=
n

i=1
_
T
2
q
i
+
T
1
q
i
+
T
0
q
i
_
q
i
= 2T
2
+ T
1
+ 0
(1) h = 2T
2
+ T
1
T
1
(T V )
= 2T
2
+ T
1
T
2
T
0
+ V
= T
2
T
0
+ V = T

+ V

The energy T

+ V

is constant for any conservative system but


it is not equal to total energy.
Natual system :
A natual system is a consevative system.
Proof :
A system is said to be natural system if
67
(i) it is described by standard form of holonomic Lagrangian equa-
tion.
(ii) the Lagrangian function L is not an explicit function of time.
(iii) any constraint equation can be expressed in the dierential form
n

i=1
a
ji
dq
i
= 0
(iv) the K.E is expressed as homogeneous quadratic functio of qs.
To prove Jacobi integral = Total energy for the natural system.
T =
1
2
n

i=1
m

j=1
m
ij
q
i
q
j
= T
2
T
0
= 0; T
1
= 0
We know that
L = T V, L = T
2
V
L
q
i
=
T
2
q
i
n

i=1
L
q
i
q
i
=
n

i=1
T
2
q
i
q
i
= 2T
2
[ by the above Book work]
h =
n

i=1
L
q
i
q
i
L
= 2T
2
(T
2
V )
= T
2
+ V
Jacobi integral = Total evergy.
Example : 1 Suppose a mass - spring system is attached to a
frame which is translating with a uniform velocity v
0
as shown
in gure. Let
0
be the unstressed spring length and use the
elongation x as the generalized co-ordinate. Find the Jacobi
integral for the system.
68
Solution : Let m be the mass of the body. Let x be the displacement
of the body from the relating postion at time t .
0
v
k
m
0
x + l
Velocity of the body = x
Velocity of the system = v
0
+ x
The K.E of the system =
1
2
m(v
0
+ x)
2
T =
1
2
m
_
v
2
0
+ x
2
+ 2v
0
x
_
T
2
=
1
2
m x
2
T
1
=
1
2
mv
0
x
T
0
=
1
2
mv
2
0
Let V be the potntial Energy of the system.
V =
1
2
kx
2
L = T V
=
1
2
mv
2
0
+
1
2
m x
2
+ mv
0
x =
1
2
kx
2
L is independent of t
Hence the system is conservative.
Jacobian integral h = T
2
T
0
+ V
h =
L
x
x L
= (m x + mv
0
) x
_
1
2
mv
2
0
+
1
2
m x
2
+ mv
0
x
2

1
2
kx
2
_
69
= m x
2
+ mv
0
x
1
2
mv
2
0

1
2
m x
2
mv
0
x
2
+
1
2
kx
2
=
1
2
m x
2

1
2
mv
2
0
+
1
2
kx
2
= T
2
T
0
+ V
Example 2 : A small tabe, bent in the form of a circle of
radius r, rotates about a vertical diameteer with a constant
angular velocity . A particle of mass m can slide without
friction inside the tube. At any given time, the conguration
of the system is specied by the angle which is measured
from the upward vertical to the line connecting the center O
and the particle. Find the Jacobi integral.
Solution :
w
g
O
q
r
m
Let O be the center of the tube. The transformation equations
relating the generalized co-ordinate and the position (x, y, z) of the
particle.
x = r sin cos t
y = r sin sin t
z = r cos
70
AP
OP
= sin AP = r sin
OA
OP
= cos OA = r cos
where r is the radius of the tube.
Let T be the K.E of the particle of mass m, V is the P.E of the
particle
T =
1
2
m
_
r
2

2
+ r
2

2
sin
2

_
=
1
2
mr
2
_

2
+
2
sin
2

_
V = mgr cos , T
2
=
1
2
mr
2

2
, T
1
= 0, T
0
=
1
2
mr
2

2
sin
2

L = T V =
1
2
mr
2
_

2
+
2
sin
2

_
mgr cos
L is independent of t.
Hence the system is conservative Jacobi method.
h =
n

i=1
L
q
i
q
i
L
=
L

L
=
_
mr
2


_
1
2
mr
2

2
+
1
2
r
2

2
sin
2
mg cos
_
=
1
2
mr
2

1
2
r
2

2
sin
2
+ mg cos
= T
2
T
0
+ V.
Example 3 :
Two particles, each of mass m, are connected by a rigid mass-
less rod of length . The particles are supported by knife
edges placed perpendicular of the rod. Assuming that all
moion is conned to the horizontal xy plane, nd the Jacobi
integral.
Solution: Let (x
1
, y
1
), (x
2
, y
2
) be the co-ordinate of the two ends of
71
the rod at time t.
Let (x, y) be the co-ordinates of the cm and we have holonomic con-
straint
(x
1
x
2
)
2
+ (y
1
y
2
)
2
=
2
(1)
We know that
tan t =
y
2
y
1
x
2
x
1
y
2
y
1
= (x
2
x
1
) tan t (2)
x
1
= x

2
cos t
x
2
= x +

2
cos t
y
1
= y

2
sin t
y
2
= y +

2
sin t
x
1
+ x
2
= 2x
y
1
+ y
2
= 2y
Let T be the K.E. of the system. Let V be the P.E. of the system.
V = 0
T =
1
2
m
_
x
2
1
+ y
2
1
_
+
1
2
m
_
x
2
2
+ y
2
2
_
x
1
= x +

2
sin t
x
2
= x

2
sin t
y
1
= y

2
cos t
y
2
= y +

2
cos t
x
2
1
+ x
2
2
+ y
2
1
+ y
2
2
= 2
_
x
2
+ y
2
_
+

2
2
cos

2
= 2
_
x
2
+ y
2
_
+ 2

2
4
72
T = m
_
x
2
+ y
2
_
+
m
2

2
4
V
0
= 0, T
2
= m x
2
+ m y
2
T
0
=
m
2

2
4
,
h = T
2
T
0
+ V
L = T V = T 0 = T
L is independent of t.
cos t x + sin t y = 0
a
ji
t = 0
The system is conservative.
Jacobi integral h =
n

i=1
L
q
i
q
i
L
=
L
x
x +
L
y
y L
= (2m x) x + (2m y) y
_
m x
2
+ m y
2
+
m
2

2
4
_
= m x
2
+ m y
2

1
4
m
2

2
.
These result show that the toal mass is M.I about the cente of m
2
/2.
Orthogonal System :
Orthogonal system is an natural system. A natural system is said
to be orthogonal if
i) It is described by standard holonomic Lagrangian equation.
ii) The Lagrangian function L is not an explicit functio of time.
iii) The K.E contains only q
2
i
and no cross products in the q

s.
Liouvilles system :
A system is said to be Liouvilles system if
73
i) It is described by standard holomic Lagranges equation.
ii) The Lagranges function is not an explicit function of time.
iii) Any constraint equation can be expressed in the dierential form
n

i=1
a
ji
dq
i
= 0, j = 1, 2, 3, , m
iv) The K.E. Tand P.E. V are given by
K.E.T =
1
2
f
n

i=1
m
i
(q
i
) (q
i
)
2
where m
i
(q
i
) > 0
P.E.V =
1
f
n

i=1
v
i
(q
i
)
f =
n

i=1
f
i
(q
i
> 0)
Liouvilles system special case of orthogonal system.
Book Work : Liouvilles system special case of orthogonal system
(OR)
With usual notation show that the following system is separable (or)
Reduce the orthogonal system to quadrature. (Or)
Explain one of the Quadratic system to be orthogonal system.
T =
1
2
f
n

i=1
q
2
i
V =
1
f
n

i=1
v
i
(q
i
)
f =
n

i=1
f
i
(q
i
> 0)
Proof : Lagranges equation is in the form
d
dt
_
L
q
i
_

L
q
i
= 0
74
f = f
1
(q
1
) + f
2
(q
2
) + + f
i
(q
i
) + + f
n
(q
n
)
L = T V
d
dt
_
(T V )
q
i
_

(T V )
q
i
= 0
d
dt
_
T
q
i
_

T
q
i
+
V
q
i
= 0
d
dt
(f q
i
)
1
2
f
i
q
i
n

i=1
q
2
i
+
_
1
f
v
i
q
i

1
f
2
f
i
q
i
n

i=1
v
i
(q
i
)
_
= 0
d
dt
(f q
i
)
1
2
f
i
q
i
n

j=1
q
2
j
+
1
f
v
i
q
i

v
f
f
i
q
i
= 0 (1)
This is a natural system.
T + V = h; q
i
= q
j
1
2
f
n

j=1
q
2
j
+ V = h
1
2
n

j=1
q
2
j
=
h V
f
(1)
d
dt
(f q
i
)
h V
f
_
f
i
q
i
_
+
1
f
_
v
i
q
i
_

V
f
_
v
i
q
i
_
= 0
d
dt
(f q
i
)
h
f
_
f
i
q
i
_
+
1
f
_
v
i
q
i
_
= 0
Multiply 2f q
i
on both sides
2f q
i
d
dt
(f q
i
) 2f q
i
_
h
f
__
f
i
q
i
_
+
2f q
i
f
_
v
i
q
i
_
= 0
2f q
i
_
d
dt
(f q
i
)
_
2hf q
i
_
f
i
q
i
_
+ 2 q
i
_
v
i
q
i
_
= 0
d
dt
_
f
2
q
2
i
_
2h
_
dq
i
dt
__
f
i
q
i
_
+ 2 q
i
_
v
i
q
i
_
= 0
d
dt
(f q
i
)
2
2h
_
dq
i
dt
__
f
i
q
i
_
+ 2 q
i
_
v
i
q
i
_
= 0
75
d
dt
(f q
i
)
2
2h
_
dq
i
dt
__
f
i
q
i
_
+ 2
_
v
i
q
i
__
dq
i
dt
_
= 0 (2)
d
dt
(f q
i
)
2
2h
_
f
i
t
_
+ 2
_
v
i
t
_
= 0
d
dt
(f q
i
)
2
= 2h
_
f
i
t
_
2
_
v
i
t
_
= 2
d
dt
(hf
i
v
i
)
T + V =
1
2
f
n

i=1
q
2
i
+ V = h
1
2
f
n

i=1
q
2
i
+ 2
n

i=1
v
i
(q
i
) = h
Multiy 2f on both sides,
f
2
n

i=1
q
2
i
+ 2
n

i=1
v
i
(q
i
) 2fh = 0 (3)
(2)
d
dt
_
f
2
i
q
2
i
_
= 2
_
d
dt
(hf
i
v
i
)
_
_
d
dt
_
f
2
q
2
i
_
= 2
_ _
d
dt
(hf
i
v
i
)
_
f
2
q
i
= 2 (hf
i
v
i
) + 2c
i
q
2
i
=
2
f
2
(hf
i
+ c
i
v
i
)
Let 2c
i
= f
2
q
2
i
2fh
i
+ 2v
i
.
2
n

i=1
c
i
= 0
n

i=1
c
i
= 0 [by (3)]
Here c

i
s and h together comprise n - independent constant of mo-
tion.
76
q
2
i
=
2
f
2
i
[(hf
i
v
i
) + c
i
]
q
i
=
_
2 [(hf
i
v
i
) + c
i
]
f
dq
i
dt
=
_
2 [(hf
i
v
i
) + c
i
]
f
dq
i
_
2 [(hf
i
v
i
) + c
i
]
=
dt
f
dq
1
_
2 [(hf
1
v
1
) + c
1
]
=
dq
2
_
2 [(hf
2
v
2
) + c
2
]
=
=
dq
n
_
2 [(hf
n
v
n
) + c
n
]
=
dt
f
= d (4)
Hence required system reduced to quadratures.
Note:
Replacing dq
i
by
_
m
i
(q
i
)dq
i
T =
1
2
f
n

i=1
m
i
(q
i
) q
2
i
where m
i
(q
i
) > 0
V =
1
f
n

i=1
v
i
(q
i
)
i
; f =
n

i=1
f
i
(q
i
) > 0
A natural system of this type is Liouvilles system.
The equation (4)
dq
1
_
Q
1
(q
1
)
=
dq
2
_
Q
2
(q
2
)
= =
dq
n
_
Q
n
(q
n
)
=
dt
f
= d
Q
i
(q
i
) =
2
m
i
(hf
i
v
i
+ c
i
) , i = 1, 2, 3, , n
f
1
dq
1
_
Q
1
(q
1
)
=
f
2
dq
2
_
Q
2
(q
2
)
= =
f
n
dq
n
_
Q
n
(q
n
)
=
f dt
f
77
f
1
dq
1
_
Q
1
(q
1
)
+
f
2
dq
2
_
Q
2
(q
2
)
+ +
f
n
dq
n
_
Q
n
(q
n
)
=
(f
1
+ f
2
+ + f
n
)
f
dt
=
f
f
dt = dt
dt =
n

i=1
f
i
dq
i
_
Q
1
(q
i
)
= t +
1
, where
1
is constant.
Show that spherical pendularn problem can be solved com-
pletly by quadratures.
Solution :
T =
1
2
m
_

2
+
2

2
sin
2

_
V = mg cos
where m is the mass of the particle.
h = T + V, L = T V
The system is conservative and natural.
h =
1
2
m
_

2
+
2

2
sin
2

_
+ mg cos
L =
1
2
m
_

2
+
2

2
sin
2

_
mg cos
L is an independent on , is ignorable.
L

= a constant =
1
2
m
2
2

sin
2
=

=
2
m
2
sin
2

(1) h =
1
2
m
2

2
+
1
2
m
2

2
sin
2

m
2

4
sin
4

+ mg cos
=
1
2
m
2

2
+

2
2m
2
sin
2

+ mg cos
78
Multiply by 2m
2
sin
2
,
m
2

4
sin
2

2
+ 2
2
= 2m
2
sin
2
(h mg cos )

=
_
2m
2
sin
2
(h mg cos ) 2
2
m
2
sin

=
d
dt
.
This values apply d

0
m
2
sin
_
2m
2
sin
2
(h mg cos ) 2
2
d =
t
_
t
0
dt = t t
0
And

m
2
sin
2

d
dt
=

m
2
sin
2

d =

m
2
sin
2

dt
(1)

m
2
sin
2

dt =
m
2
sin d
2m
2
sin
2

m
2
sin
(h mg cos )
2
=
d
sin
_
2m
2
sin
2
(h mg cos )
2

0
d =

0
d
sin
_
2m
2
sin
2
(h mg cos )
2

0
=

0
d
sin
_
2m
2
sin
2
(h mg cos )
2
where
0
= (t
0
) .
Now we have obtained the required four constraints of motion
79
namely , h, t
0
,
0
.
Discuss probem of the spherical pendulum considering it as
a Liovilles system.
Solution : We know that
T =
1
2
m
_

2
+
2

2
sin
2

_
V = mg cos
For Liovilles system
T =
1
2
f
_
n

i=1
m
i
(q
i
) q
2
i
_
V =
1
f
n

i=1
v
i
(q
i
)
i
1
2
(f

+ f

) = m

2
+ m

2
=
1
2
_
m
2

2
+ m
2
sin
2

2
_
(f

+ f

) M

= m
2
(f

+ f

) M

= m
2
sin
2

=
1
sin
2

, M

= 1, M

=
1
sin
2

(f

+ f

) v = v
1
() + v ()
_
M
2
sin
2

_
mg cos = v
1
()
v

= m
2
g
3
sin
2
cos
v
1
() = m
2
g
3
sin
2
cos
f

+ f

= m
2
sin
2

= m
2
sin
2
[ f

= 0]
We know that

i
(q
i
) =
2
M
i
(hf
i
v
i
+ c
i
) (i = 1, 2, , n)

=
2
M

(hf

+ c

)
80
=
2
M

_
hm
2
sin
2
m
2
g
3
sin
2
cos

= 2 sin
2

_
M
2
sin
2
(h mg cos ) + c

=
2
M

[hf

+ c

]
= 2 [h(0) 0 + c

]
= 2c

Using
f
2

2
i
= 2 [hf
i
(g
i
) v
i
(q
i
) + c
i
]
n

i=1
c
i
= 0
= 2 [hf

+ c

]
L

= m
2
sin
2
=
(f

+ f

)
2

2
= 2 [h(0) 0 + c

]
m
2
sin
2
= 2c

2c

= 2c

= 2
2
2c

= 2c

=
_
m
2

sin
2

_
2
=
2
n

i=1
_
f
i
dq
i
_

i
(q
i
)
= t +
1

0
f

d
_

()
= t t
0

0
m
2
sin
2

()
d = t t
0
We have
_
dq
1
_

1
(q
1
)

_
dq
i
_

j
(q
j
)
=
j
, j = 1, 2, 3, , n
81

0
d
_

0
()
=

0
d
_
2c

= 2c

d
_

0
()
=
0
Now we have obtained four required constraint c

, c

,
0
.
Book work :Small Oscillations
Obtain the equation of the natural system by given a small
motion about its equilibrium position. OR
Obtain the equation of natural system in matrices form as
m q + kq = 0
Proof :
Let the natural system be given by n independent generalized co-
ordinates q
1
, q
2
, q
3
, ., q
n
. Let us assume all the q

s all measurable
from a position of equilibrium.
Consider small motion about this equilibrium position.
Let v
0
be the reference point is zero. The P.E. can be written in
the form
V =
1
2
n

i=1
n

j=1
_

2
v
q
i
q
j
_
0
q
i
q
j
= 0.
Neglecting terms of higher p order terms the second in the q

s we obtain
V =
1
2
n

i=1
n

j=1
_

2
v
q
i
q
j
_
0
q
i
q
j
V =
1
2
n

i=1
n

j=1
k
ij
q
i
q
j
where k
ij
=

2
v
q
i
q
j
is the stiness co-ecient.
V is the homogenous as quoctratic of the q

s for small motion near.


Small motion near a positon of equilibrium.
Let us assume the system consists N - particles with caresian Co-
82
ordinates x
1
, x
2
, .., x
3N
then the K.E of the system is given by
T =
1
2
n

i=1
n

j=1
m
ij
q
i
q
j
where m
ij
= m
ji
=
3N

k=1
m
k
_
x
k
q
i
__
x
k
q
j
_
L = T V
=
1
2
n

i=1
n

j=1
m
ij
q
i
q
j

1
2
n

i=1
n

j=1
k
ij
q
i
q
j
(1)
The standard Lagranges equation for a holomic system is
d
dt
_
L
q
i
_

L
q
i
= 0, i = 1, 2, 3, , n
(1)
n

j=1
m
ij
q
j
+
n

j=1
k
ij
q
j
= 0
m
j
q
j
+ k
ij
q
j
= 0
m q + kq = 0
This is the equation of natural system
Memory point : i) For a spherical pendulum
L = T V
T =
1
2
m
_

2
+
2

2
sin
2

_
V = mg sin
ii) For double pendulum
v
1
=

, v
2
=
_

2
+
2

2
+ 2

cos ( )
T =
1
2
m
_
v
2
1
+ v
2
2
_
83
=
1
2
m
2

2
+
1
2
m
_

2
+
2

2
+ 2

cos ( )
_
V = mg (2 cos + cos )
iii) For a small tube bent is
T =
1
2
m
_
r
2

2
+ r
2

2
sin
2

_
L = mgr cos
iv) For mass storing system
T =
1
2
m r
2

2
+
1
2
m

r
P
m

C
=
1
2
m

r
2
P
+
1
2
m

2
+

r
P

r
P
= a,

= r
2
+ r
2

r
P
m

= ma
_
r sin ( t) + r

cos ( t)
_
T =
1
2
ma
2

2
+
1
2
m
_
r
2
+ r
2

2
_
+ma
_
r sin ( t) + r

cos ( t)
_
V =
1
2
k (r r
0
)
2
v) For Keplers Probem
T =
1
2
_
r
2
+ r
2

2
_
V =

r
, L = T V
R = L

vi) Two particles each of masses m nd Jjacobi integral.


(x
1
x
2
)
2
+ (y
1
y
2
)
2
=
2
tan t =
y
2
y
1
x
2
x
1
(y
2
y
1
) = (x
2
x
1
) tan t
84
UNIT III
Hamiltons Equations
Stationary Value :
To nd the stationary values of the function f = z subject to
constraints

1
= x
2
+ y
2
+ z
2
4 = 0

2
= xy 1 = 0
combined force F = f +
1

1
+
2

2
.
Solution:
Let augmented function be
F = z +
1

1
+
2

2
= z +
1
_
x
2
+ y
2
+ z
2
4
_
+
2
(xy 1)
F
x
= 2x
1
+
2
y = 0 (1)
F
y
= 2y
1
+
2
x = 0 (2)
F
z
= 1 + 2z
1
= 0 (3)
F

1
= x
2
+ y
2
+ z
2
4 = 0 (4)
F

2
= xy 1 = 0 (5)
(2)
(1)

2y
2x
=
x
y
x
2
= y
2
x = y y = x
(5) xx 1 = 0
x
2
1 = 0
x
2
= 1
85
x = 1
If x = 1, y = 1 and if x = 1, y = 1
(4) x = 1, y = 1 1 + 1 +z
2
4 = 0
z
2
= 2 z =

2
Here solving the equation, we have stationary at
_
1, 1,

2
_
,
_
1, 1,

2
_
,
_
1, 1,

2
_
and
_
1, 1,

2
_
.
(3) 1 + 2z
1
= 0
1
=
1
2

2
if z =

2
If z =

2,
1
=
1
2

2
,
1
=
1
2

2
(2) 2 (1)
1
2

2
+
2
= 0

2
=
1

2
,
2
=
1

2
=
1

2
It is seen that
_
1, 1,

2
_
,
_
1, 1,

2
_
are the constraints of maxi-
mum oints and the other two points are minimum points.
Euler Lagrage equation or stationary values a denite inte-
gral.
Solution : Suppose we wish to nd the stationary of
I =
x
1
_
x
0
f [y (x) , y

(x) , x] dx
where y

(x) =
dy
dx
and the limits are xed.
Let us assume that derivatives f (y, y

, x) has two continuous deriva-


tives in each of its arguments. We have to nd a function y

(x) which
86
gives a stationary value for I weve
y (x) = y

(x) + y (x)
where y (x) is small variation in y. But small variation
y = (x)
y (x) = (x)
y (x) = y

(x) + 2 (x)
where (x) is a arbitary function having the required smoothness and
is an parameter which does not independent of x.
Hence for any given (x) we can consider the varied curve y to be
function of and x.
i.e., y (, x) = y

(x) + (x) .
Let us assume the variation y = 0 at the end points
2 (x
0
) = 0 and (x
1
) = 0
y (x
0
) , y (x
1
) are xed.
We see that the integral I is a function of only for any given
(x) .
I having a stationary value y

(x)
I =
_
dI
d
_
=0
= 0
x
0
, x
1
are independent on
y (x, ) = y

(x) + (x)
y

= (x) ;
y

(x)
87
I =
x
_
x
0
f (y, y

, x) dx
dI
dx
=
x
_
x
0
_
f
y
y

+
f
y

x
_
dx = 0
x
_
x
0
f
y
(x) dx +
x
_
x
0
f
y

(x) dx = 0
Since (x
0
) = 0, (x
1
) = 0, we have
x
_
x
0
f
y
(x) dx +
_
(x)
f
y

_
x
x
0

x
_
x
0
(x)
d
dx
_
f
y

_
dx = 0
x
_
x
0
f
y
(x) dx +
_
(x)
f
y

_
x
x
0

x
_
x
0
(x)
d
dx
_
f
y

_
dx = 0
x
_
x
0
f
y
(x) dx
x
_
x
0
(x)
d
dx
_
f
y

_
dx = 0
Since (x) is arbitary we have
f
y

d
dx
_
f
y

_
= 0

d
dx
_
f
y

f
y
= 0
This equation is known as Euler- Lagrages equation for any curve
y = y

(x) which result in a stationary value of I.


Branchistochrone Probem :
It is a nd a curve y (x) between origin O and the point (x
1
, y
1
)
such that a particle starting from rest at O and sliding down the
curve without friction under the inuence of uniform gravity eld in a
minimum time to reach the end of the curve.
88
Solution :
The vertical distance described by the particle to corresponding to
the point (x
1
, y
1
) is x with velocity.
Let P be the position of the particle at time with velocity v.
From principle of conservation of energy we obtain
mgx =
1
2
mv
2
v =
_
2gx
Let t be the time taken to reach the point (x
1
, y
1
) is found by noting
rst that an innitesimal path element ds is given by from O
ds =
_
1 + y
2
dx
t =
s
_
0
ds
v
=
x
1
_
0
_
1 + y
2

2gx
dx.
Let f (y, y

, x) =

1 + y
2
2gx
. The Euler Lagrang equation is
F
y

d
dx
_
F
y

_
= 0 (1)
F
y
= 0
0
d
dx
_
F
y

_
= 0
F
y

= C where C is constant
F
y

=

y

_
_

1 + y
2
2gx
_
_
= C
89

1
2

1 + y
2
2gx

2y

2gx
= C
y

2gx
_
1 + y
2
= C
y
2
= C
2
_
2gx
_
1 + y
2
_
= 2C
2
gx + 2gxC
2
y
2
=
_
1 2gxC
2
2gxC
2
_
y
2
2gxC
2
y
2
= 2C
2
gx y
2
_
1 2gxC
2
_
= 2C
2
gx
y
2
=
2C
2
gx
(1 2gxC
2
)
y

2C
2
gx
(1 2gxC
2
)
Let x = a (1 cos ) where a =
1
4gC
2
.
y

_
2gC
2

1
4gC
2
(1 cos )
1 2gC
2
1
4gC
2
(1 cos )

_
1
2
(1 cos )
1
1
2
(1 cos )
=

_
1
2
(1 cos )
1 2gC
2
1
4gC
2
(1 cos )
_
1 cos
2 cos
=
_
1 cos
1 + cos
=

_
2 sin
2

2
2 cos
2

2
dy
dx
=
sin

2
cos

2
dy =
sin

2
cos

2
dx
dy =
sin

2
cos

2
a sin d
90
=
sin

2
cos

2
a2 sin

2
cos

2
d
= 2a sin
2

2
d
_
dy =
_
2a sin
2

2
d = a
_
(1 cos ) d
y = a ( sin ) + c
At x = y = 0, = 0 c = 0.
Path is y = a( sin ), x = a (1 cos ) which is cycloid. The
constant a is choosen t the poth always through the point (x
1
, y
1
).
By comparing other paths through O and (x
1
, y
1
) we can prove only
the cycloid path has minimum time.
Note:
y
2
1 y
2

y
_
1 + y
2
= c
1

y
_
1 + y
2
= c

_
1 + y
2

y =
1
c
Taking square on both sides
_
1 + y
2
_
( y) =
1
c
2
= c
1
y
2
=
c
1
y
1 =
c
1
+ y
y
y

=
_
c
1
+ y
y
take y

=
_
c
1
+ y
y
dy
dx
=
_
c
1
+ y
y

c
1
+ y
dy = dx (1)
91
Take y = c
1
sin
2

2
, dy = c
1
2 sin

2
cos

2
d

2
Geodesic Problem :
Denition :
The Problem of nding the shortest path between two points in a
given space is called geoclesic.
The shortest path between two points is given space is the path
which forms a greaest circle which is the geodesic.
To prove this let us consider the equation of the problem.
Find the geodesic for the spherical surface.
(or)
Find the path of minimum length between two given points on the
two dimensional surface of a sphere of radius r
Let the spherical co-ordinate, (, ) of the point of spherical surface.
Proof:
Let us use the spherical co-ordinate (, ) as variable since r is a
constant.
We know that
x = r sin cos
y = r sin sin
z = r cos
The dierential elements of length ds is given by
ds
2
= r
2
d
2
+ r
2
sin
2
d
2
= r
2
d
2
_
1 + sin
2
_
d
d
_
2
_
ds = r

1 + sin
2
_
d
d
_
2
d
s = r

0
_
1 +
2
1
sin
2
d where

=
d
d
.
92
Let f (,

, ) =
_
1 +
2
sin
2
.We want to nd the shortest dis-
tance of the path P and Q.
we have to nd the stationary value of Euler - Lagrangian equa-
tion is
f
y

d
dx
_
f
y

_
= 0.
But here f is a function of (,

, ) .
The new Eulers Lagrangian equation for thus problem is
f


d
d
_
f

_
= 0.
But f =
_
1 +
2
sin
2
.

= 0,
f

=
1
2
_
1 +
2
sin
2

sin
2

Using (1) Eulers Lagrangian equation


d
d
_
f

_
= 0
f

= c
F

sin
2

_
1 +
2
sin
2

= c

sin
2
= c
_
1 +
2
sin
2

Squaring and rearranging we get


sin
4

2
= c
2
_
1 +
2
sin
2

_
sin
2

2
_
sin
2
c
2
_
= c
2

2
=
c
2
sin
2

_
sin
2
c
2
_

=
c
sin
_
sin
2
c
2
=
_
c
sin
_
sin
2
c
2
d
93
=
_
c
sin
2

_
1
c
2
sin
2

d
=
_
c
sin
2

1 c
2
cosec
2

d
=
_
c cosec
2

1 c
2
cosec
2

d
= c
_
cosec
2

_
1 c
2
(1 + cot
2
)
d
= c
_
d (cot )

1 c
2
c
2
cot
2

= c
_
d (cot )
c

_
1 c
2
c
2
_
cot
2

= cos
1
_
_
_
_
_
_
cot

_
1 c
2
c
2
_
_
_
_
_
_
_
+
0
= cos
1
_
_
_
_
_
_
cot

_
1 c
2
c
2
_
_
_
_
_
_
_
= cos
1
_
c cot

1 c
2
_
cos (
0
) =
c

1 c
2
cot
=
c cot

1 c
2
cos cos
0
sin sin
0
=
c

1 c
2
cos
sin
Multiply by r sin ,
r sin cos cos
0
+ r sin sin sin
0
=
c

1 c
2
cos
sin
r sin
=
c

1 c
2
r cos
94
The above equation becomes
xcos
0
+ y sin
0
=
c

1 c
2
z
Then the equation of the form ax + by + cz = 0 . This represents
the plane through the origin. This plane intersects the sphere along a
great circle which is geodesic.
Here
0
and c are two constants which are adjustable such that the
greats circle will pass through P and Q.
General Eulers Lagrangian equation :
Let
I =
x
1
_
x
0
f (y
1
, y
2
, , y
n
, y

1
, y

2
, , y

n
, x) .
We have found a necessary condition for a stationary value of I.x
0
, r
are xed limits.
State and prove Hamiltons principle :
Variational principle or integration principle[Integrated form
of DAlemberts principle]
Statement :
The actual path in conguration space followed by a holonomic
dynomical system during the xed interval t
0
to t
1
is such that the
integral I =
t
1
_
t
0
Ldt is the stationary value with respect to path vari-
ation which vanish at the end points I = 0. Here L is a function
L(q, q

, t) .q is a n dimensional point q
1
, q
2
, , q
n
.
Proof :
Let r
1
, r
2
, , r
N
be the position vectors of the N particles.
By the form of D Alemberts principle
r
N

i=1
_

F
i
m
i

r
i
_
r
i
= 0
95
where

F
i
is the applied force acting on the i
th
particle.
N

i=1

F
i
r
i

i=1
m
i

r
i
r
i
= 0 (1)
Virtual work of A.F.workdone = W =
N

i=1

F
i
r
i
.
Now variation in K.E.
T =
_
1
2
N

i=1
m
i

r
i
_
=
1
2
N

i=1
m
i
2

r
i

r
i
T =
N

i=1
m
i

r
i

r
i
(2)
But
d
dt
_
N

i=1
m
i

r
i
r
i
_
=
N

i=1
m
i

r
i
r
i
+
N

i=1
m
i

r
i

r
i
(3)
where
d
dt
(r
i
) =

r
i
= W + T
From (2) and (1)
d
dt
_
N

i=1
m
i

r
i
r
i
_
=
N

i=1
m
i

r
i
r
i
+
N

i=1
m
i

r
i

r
i
= T +
N

i=1

F
i
r
i
T + W =
d
dt
_
N

i=1
m
i

r
i
r
i
_
( equation (3))
(T + W) dt = d
_
N

i=1
m
i

r
i
r
i
_
96
t
1
_
t
0
(T + W) dt =
t
1
_
t
0
d
_
N

i=1
m
i

r
i
r
i
_
=
_
N

i=1
m
i

r
i
r
i
_
t
1
t
0
= 0
Now let us assume that the conguration of the system is xed at the
time t
0
to t
1
implying the variation r
i
are zero at these times.

i=1
_
m
i

r
i
r
i
_
t
1
t
0
= 0

t
1
_
t
0
(T + W) dt = 0 (4)
For a given virtual displacement and a time the value of T and
W are independent of these Co-ordinates system.
This case of generalized forces equation (4) can be written in the
form
t
1
_
t
0
_
T +
N

i=1
Q
i
q
i
_
dt = 0 (5)
Version of Hamiltons principle :
Let us assume all the applied forces and derivable from a potentaial
function V (q, t) .
W = V
(4)
t
1
_
t
0
(T + W) dt =
t
1
_
t
0
(T V ) dt = 0
=
t
1
_
t
0
(T V ) dt = 0
=
t
1
_
t
0
Ldt = 0
97
=
t
1
_
t
0
Ldt
I = 0
t
1
_
t
0
Ldt = 0 is Hamiltons principle.
Hamilton principle and lagrangian equation are equiva-
lent.
(Or)
Derive the lagrangian equation from Hamilton principle.
Solution :
By Hamiltons principle
I = 0
where I =
t
1
_
t
0
L(q, q, t) dt.
Hence L(q, q, t) corresponds to f(y, y, x). Assuming q

s are inde-
pendent and using Euler Lagrangian equation takes the form
L
q
i

d
dt
_
L
q
i
_
= 0
d
dt
_
L
q
i
_

L
q
i
= 0 for all i = 1 to n.
Hamiltons equation or Hamiltons cononical equation of
motion :
Consider a holonomic system which can be described by the stan-
dard form by of lagrangian equation, namely.
d
dt
_
L
q
i
_

L
q
i
= 0 , i = 1 to n. (1)
The generalized momentum conjugate to q
i
is given by
p
i
=
L
q
i
(2)
98
(1)
d
dt
(p
i
)
L
q
i
= 0
d
dt
(p
i
) =
L
q
i
L
q
i
= p
i
(3)
Now, let us dene the Hamiltons function H (q, p, t) for the system
as
H (q, p, t) =
n

i=1
p
i
q
i
L(q, q, t) (4)
H is an explict function of q

s and p

s and t
H (q, p, t) =
N

i=1
p
i
q
i
L(q, q, t)
H =
n

i=1
H
q
i
q
i
+
n

i=1
H
p
i
p
i
+
H
t
t
=
n

i=1
p
i
q
i
+
n

i=1
H
p
i
p
i

i=1
L
q
i
p
i

i=1
L
q
i
q
i

L
t
t
n

i=1
H
q
i
q
i
+
n

i=1
H
p
i
p
i
+
H
t
t =
n

i=1
p
i
q
i
+
n

i=1
q
i
p
i

i=1
p
i
q
i

L
t
t
Equating co-ecients on both sides
p
i

H
p
i
= q
i
q
i

H
q
i
= p
i
;
L
t
=
H
t
H
p
i
= q
i
;
H
q
i
= p (5)
L
t
=
H
t
The 2n rst order Lagrangian equations is given the set of equation
(5) is called Hamilton equationn of motion.
Hamitons funciton is generalized quadratic in the ps or
99
the form of Hamiltonss function.
Proof :
We have
T
q
i
= p
i
=
n

j=1
m
ij
(q, t) q
j
+ a
i
(q, t)
=
n

j=1
m
ij
q
j
+ a
i
n

i=1
p
i
q
i
=
n

i=1
n

j=1
m
ij
q
i
q
j
+
n

i=1
q
i
q
i
= 2T
2
+ T
1
We know that
H =
n

i=1
p
i
q
i
L(q, q, t)
=
n

i=1
p
i
q
i
(T V )
= 2T
2
+ T
1
T
1
T
2
T
0
+ V
= T
2
T
0
+ V
T
2
=
1
2
n

i=1
n

j=1
m
ij
q
i
q
j
.
This can be put in the matrix form as follows
T
2
=
1
2
m
T
q q.
But q = b (p a) bm
1
. Both b, m are symmetric matrix.
b
T
= b, m = m
T
T
2
=
1
2
b
T
(p a)
T
bb
1
(p a)
=
1
2
b (p a) (p a)
T
100
Expanding this
T
2
=
1
2
n

i=1
n

j=1
b
ij
p
i
p
j

i=1
n

j=1
b
ij
a
i
p
i
+
1
2
n

i=1
n

j=1
b
ij
a
i
a
j
T
0
, V are function of q

s and t adding we have


H (q, p, t) =
1
2
n

i=1
n

j=1
b
ij
p
i
p
j

i=1
n

j=1
b
ij
a
i
p
i
+
1
2
n

i=1
n

j=1
b
ij
a
i
a
j
T
0
+V
Grouping term by their degree
H = H
2
+ H
1
+ H
0
H
2
=
1
2
n

i=1
n

j=1
b
ij
p
i
p
j
H
1
=
n

i=1
n

j=1
b
ij
p
j
a
i
H
0
=
1
2
n

i=1
n

j=1
b
ij
a
i
a
j
T
0
+ V
Hence the Hamiltons function is in general quadratic in ps.
The Hamilton function of scleronomic system is equal to
toal energy.
Let us consider a system of cartesian co-ordinates do not certain
time t explicity.
It follows that as all zero and T = T
2
.
In sceleronomic system T
0
= T
1
= 0.
H
2
= T
2
T
1
= T
2
and H
1
= 0; H
0
= V
i.e., H = T + V
101
The Hamiltons function of sceleronomic system is equal to T.K.
H (q, p, t) =
1
2
n

i=1
n

j=1
b
ij
p
i
p
j
+ V (q, t)
Non - holonomic system has a constant Hamiltons func-
tion.
Solution :
Let us have a conservative holonomic system we have H = H (q, p, t) .
dH
dt
=

H =
n

i=1
__
H
q
i
q
i
t
_
+
_
H
p
i
p
i
t
__
+
_
H
t
t
t
_

H =
n

i=1
__
H
q
i
q
i
_
+
_
H
p
i
p
i
__
+
H
t
Using Hamiltons canonical equation
H
p
i
= q
i
,
H
q
i
= p
i
(A)
and
H
t
=
L
t


H =
n

i=1
( p
i
q
i
+ p
i
q
i
)
L
t
=
L
t
.
If the system is conservative. L does not contain t explicity.

L
t
= 0

H = 0
H = C ( constant)
Hence the proof.
Phase space :
Hamiltons canonical eqn of motion consisting of a set of 2n rst
order equation giving q, p as function qs, ps are after considered as
components of a single vector X.
Thus the equation of motion for a standard holonomic system can
102
be written as x = X (x, t) where X is a 2n dimensional vector consist-
ing of nqs and nps.
Consider the qs as x
1
, x
2
, , x
n
and ps as x
n+1
, x
n+2
, , x
2n
.This
2n dimensional x space is called phase space.
Extended phase space :
In a non-holonomic conservative system, let us consider the variable
q
n+1
as choose a parameter q as the new independent variable. This
system consist of (n + 1)qs and corresponding (n + 1)ps.
This space is called extened phase space.
Autonomous - Non autonomous :
In a conservative system H is not explicit function of time and
conguration of motion of form x = X (x) .
A holonomic system described by equation of this form is called
autonomous where as x = X (x) is called non autonomous.
Liouvilles theorem :
The phase uid incompressibel.
Proof :
Let a holonomic system be described by n independents qs.
Let us consider a gropp of phase ps described fraectorcies in a 2n
dimensional phase space.
Considering the moving points in a small elementry volume dV =
dq
1
, dq
2
, , dq
n
; dp
1
, dp
2
, , dp
n
.
Consisting the moving points of a uid being called phase uid.
The phase velocity v of a uid. Particle is given by the 2n component
( q
i
, p
i
) these can be expresed as function of qs and ps by canonical
equation
q
i
=
H
p
i
, p
i
=
H
q
i
As a given volume elements of the phase uid moves it will ingeneral
change in it shape. But the neighbouring particles will remain close to
each other. We can show that volume of each phase uid is constant
103
during the motion.
v =
n

i=1
_

q
i
q
i
+

p
i
p
i
_
=
n

i=1
_

q
i
_
H
p
i
_
+

p
i
_

H
q
i
__
=
n

i=1
_

2
H
q
i
p
i


2
H
p
i
q
i
_
= 0
v = 0
A geometric interpretation of this results that the phase uid is
incompressible.
Keplers problem :
Use the Jacobi form of the principle of least action. Obtaing the
orbit for the Keplers problem.
Solution :
Let a particle of mass m be attracted to a xed point O by an
inverse square force.
V =
m
r
, F
r
=
m
r
2
T =
1
2
mv
2
=
1
2
m
_
r
2
+ r
2

2
_
h = T + V
=
1
2
m
_
r
2
+ r
2

2
_

m
r
(1)
By Jacobi form of principle of least action

_
_
2 (h v)ds = 0

_
_
2
_
h +
m
r
_
ds = 0 (2)
104
we have ds
2
= m
_
dr
2
+ r
2
d
2
_
_
ds
d
_
2
= m
_
_
dr
d
_
2
+ r
2
_
= m
_
r
2
+ r
2
_
ds
d
=
_
m(r
2
+ r
2
)
ds =
_
m(r
2
+ r
2
)d

_
_
2
_
h +
m
r
_
_
m(r
2
+ r
2
)d = 0

_
_
2m
_
h +
m
r
_
(r
2
+ r
2
)d = 0
where the end points this is the form of

0
f (r, r

) d = 0
where f (r, r

) =
_
2m
_
h +
m
r
_
(r
2
+ r
2
).
Applying the Eulers Lagranges equation for f (r, r

) we have
d
d
_
f
r

f
r
= 0.
The corresponding energy integral

L
q
i
q
i
L = h
f
r

f = C ( constant)
f (r, r

) =
_
2m(r
2
+ r
2
)
_
h +
m
r
_
105
f
r

=
2m
_
h +
m
r
_
2r

2
_
2m(r
2
+ r
2
)
_
h +
m
r
_
=
2m
_
h +
m
r
_
r

_
2m(r
2
+ r
2
)
_
h +
m
r
_
=

_
2m
_
h +
m
r
_
(r
2
+ r
2
)
r

f
r

f = C

_
2m
_
h +
m
r
_
(r
2
+ r
2
)
r

_
2m(r
2
+ r
2
)
_
h +
m
r
_
= C
_
2m
_
h +
m
r
_
_
r
2
_
(r
2
+ r
2
)

_
(r
2
+ r
2
)
_
= C
_
2m
_
h +
m
r
_
_
r
2
r
2
r
2
_
(r
2
+ r
2
)
_
= C

_
2m
_
h +
m
r
_
_
(r
2
+ r
2
)
r
2
= C (3)
Now
h = T + V
=
1
2
m
_
r
2
+ r
2

2
_

m
r
m
_
r
2
+ r
2

2
_
= 2
_
h +
m
r
_
m
2
_
r
2
+ r
2

2
_
= 2m
_
h +
m
r
_
(4)
r =
dr
dt
=
dr
d
d
dt
106
r =
dr
dt
= r

. =
r
r

r = r

(4) m
2
_
r
2

2
+ r
2

2
_
= 2m
_
h +
m
r
_
m
2

2
_
r
2
+ r
2
_
= 2m
_
h +
m
r
_
In (3) use this

_
2m
_
h +
m
r
_
(r
2
+ r
2
)
r
2
= C

m
2

2
(r
2
+ r
2
)
(r
2
+ r
2
)
r
2
= C

_
m
2

2
r
2
= C
m

r
2
= C

=
C
mr
2
Angular momentum is constant

2
=
C
2
m
2
r
4
To nd the equation of orbit
(3)

_
2m
_
h +
m
r
_
(r
2
+ r
2
)
r
2
= C
Squaring and rearranging we get
2m
_
h +
m
r
_
r
4
= C
2
_
r
2
+ r
2
_
C
2
r
2
= 2m
_
h +
m
r
_
r
4
C
2
r
2
107
r
2
=
2mr
4
Cdr
2
_
h +
m
r
_
r
2
=
2mr
2
C
2
_
hr
2
+ mr
C
2
2m
_
_
dr
d
_
2
=
2mr
2
C
2
_
hr
2
+ mr
C
2
2m
_
dr
d
=

2mr
2
C
2
_
hr
2
+ mr
C
2
2m
_
dr

2mr
2
C
2
_
hr
2
+ mr
C
2
2m
_
= d
_
d =
C

2m
_
dr
_
hr
4
+ mr
2
C
2
r
2
2m
=
C

2m
r
_
r
0
dr
r
2
_
h +
m
r

C
2
2m
=
r
_
r
0
d
_
m
2
C
2

1
r
_

2
m
4
C
4
+
2mh
C
2

_
m
2
C
2

1
r
_
= sin
1
_

_
_
m
2
C
2

1
r
_
_

2
m
4
C
4
+
2mh
C
2
_

_
r
r
0
= sin
1
_

_
_
m
2
C
2

1
r
_
_

2
m
4
C
4
+
2mh
C
2


2
_

_
sin
_
+

2
_
=
m
2
C
2

1
r
_

2
m
4
C
4
+
2mh
C
2
108
cos
_

2
m
4
C
4
+
2mh
C
2
=
m
2
C
2

1
r
1
r
=
m
2
C
2

_

2
m
4
C
4
+
2mh
C
2
cos
Multiplying by
C
2
m
2
we get
C
m
2
/r
= 1

1 +
2hc
2

2
m
3
cos
This is a conic with eccentricity
i.e.,

1 +
2hc
2

2
m
3
_

r
= 1 + e cos
_
To nd h :
h =
1
2
m
_
r
2
+ r
2

2
_

m
r
At r = r
0
= r
min
, =
0
= 0
m
2
_
0 +r
2

2
0
_
= h +
m
r
0
Also
mr
2
0

0
C
0
=
C
mr
2
0
(1)
m
2
_
r
2
0
C
2
m
2
r
4
0
_
= h +
m
r
0
C
2
2mr
2
0
= h +
m
r
0
h =
C
2
2mr
2
0

m
r
0
Example Given a holonomic system with Lagrangian form L =
1
2
m( x
2
1
+ x
2
2
+ x
2
3
)
mgx
0
and a constraint x
1
x
2
+ x
3
= 0. We argumented Lagrangian
function to obtain the dierential equation of motion for x
1
.
109
Solution :
L =
1
2
m
_
x
2
1
+ x
2
2
+ x
2
3
_
mgx
0
constraint:
x
1
x
2
+ x
3
= 0
= L + g
=
1
2
m
_
x
2
1
+ x
2
2
+ x
2
3
_
mgx
3
+ ( x
1
x
2
+ x
3
)
d
dt
_

x
1
_


x
1
= 0
we have
d
dt
(m x
1
+ ) = 0
m x
1
+
d
dt
= 0 (1)
d
dt
_

x
2
_


x
2
= 0
d
dt
(m x
2
) = 0
m x
2

d
dt
= 0 (2)
d
dt
_

x
3
_


x
3
= 0
d
dt
(m x
3
+ ) +mg = 0
m x
3
+ mg +
d
dt
= 0 (3)
(1), (2) and (3) are the dierential equation of motion. Itreating
are of the we have
d
dt
_

= 0
x
1
x
2
+ x
3
= 0 (4)
110
Adding (1) and (2), we get
x
1
+ x
2
= 0 x
1
= x
2
Adding (2) and (3) we get
x
2
+ x
3
= 0 x
2
= x
1
(*)
() x
1
x
3
= g
x
3
= g x
2
= g + x
1
x
1
+ x
1
+ x
1
g = 0
3 x
1
= g
x
1
=
g
3
(4) x
1
+ x
3
= x
2
x
3
= x
2
x
1
= x
1
x
1
= 2 x
1
=
2g
3
Denition : Action In mechanics action is deced as
A =
t
1
_
t
0
n

i=1
p
i
q
i
dt.
State and prove Least action or Principle of least action
The actual path of a conservative holonomic system is such that
the action is stationary with respect to varied pathes having the same
energy integral and the same end points in a q space.
Proof :
In this case we are considering a more general type of variation. In
111
this variation the point (q + q, t + t) corresponds to (q, t).
Let d indicates dierential change along individual path.
indicates variations in going from actual path to varied path
considering the small quadirlateral in the above gure.
qdt + q + dq = q + ( q + q) (dt + dt)
qdt + dq = qdt + qdt + qdt + qdt
Dividing by dt
dq = qdt + qdt + qdt
d
dt
(q
i
) = q
i
d
dt
(t) + q
i
+ q
i
d
dt
(t)
Omitting q
i
d
dt
(t)
d
dt
(q
i
) = q
i
d
dt
(t) + q
i
q
i
=
d
dt
(q
i
) q
i
d
dt
(t) (1)
Let I =
t
1
_
t
0
L(q, q, t) dt.
I =
t
1
_
t
0
Ldt =
t
1
_
t
0
Ldt +
t
1
_
t
0
Ld (t)
=
t
1
_
t
0
_
n

i=1
_
L
q
i
q
i
+
L
q
i
q
i
_
+
L
t
t + L
d
dt
(t)
_
dt
=
t
1
_
t
0
_
n

i=1
_
L
q
i
q
i
+
L
q
i
_
d
dt
(q
i
) q
i
d
dt
(t)
__
+
L
t
t + L
d
dt
(t)
_
dt
112
=
t
1
_
t
0
_
n

i=1
_
L
q
i
q
i
+
_
L
q
i
d
dt
(q
i
)
_

L
q
i
q
i
d
dt
(t)
_
+
L
t
t + L
d
dt
(t)
_
dt
Consider
d
dt
_
n

i=1
L
q
i
d
dt
(q
i
)
_
=
n

i=1
d
dt
_
L
q
i
_
d
dt
(q
i
)
n

i=1
_
L
q
i
_
d
dt
(q
i
) =
d
dt
_
n

i=1
_
L
q
i
_
d
dt
(q
i
)
_

i=1
d
dt
_
L
q
i
_
q
i
Using this value in above equation we get,
I =
t
1
_
t
0
_
n

i=1
L
q
i
q
i
+
d
dt
_
n

i=1
_
L
q
i
_
q
i
_

i=1
d
dt
_
L
q
i
_
q
i

i=1
L
q
i
q
i
d
dt
(t) +
L
t
t + L
d
dt
(t)
_
dt
=
t
1
_
t
0
d
dt
_
n

i=1
L
q
i
q
i
_
dt
t
1
_
t
0
n

i=1
_
d
dt
_
L
q
i
_

L
q
i
_
q
i
dt

t
1
_
t
0
_
n

i=1
L
q
i
q
i
L
_
d
dt
(t) dt +
t
1
_
t
0
L
t
tdt (*)
= I
1
I
2
I
3
+ I
4
where
I
1
=
t
1
_
t
0
d
dt
_
n

i=1
L
q
i
q
i
_
dt

_
n

i=1
L
q
i
q
i
_
t
1
t
0
= 0
113
I
2
=
t
1
_
t
0
n

i=1
_
d
dt
_
L
q
i
_

L
q
i
_
q
i
dt = 0
I
4
=
t
1
_
t
0
L
t
tdt = 0
I
3
=
t
1
_
t
0
_
n

i=1
L
q
i
q
i
L
_
d (t) = I
Every varied path having the same evergy integral h.
n

i=1
L
q
i
q
i
L = h
I =
t
1
_
t
0
hd (t) = h(t)
t
1
t
0
I = h(t
1
t
0
) (a)
Let us dene the action
A =
t
1
_
t
0
n

i=1
p
i
q
i
dt
=
t
1
_
t
0
n

i=1
L
q
i
q
i
dt
=
t
1
_
t
0
(L + h) dt
A =
t
1
_
t
0
Ldt +
t
1
_
t
0
hdt
= I +
t
1
_
t
0
hdt +
t
1
_
t
0
hd (t)
= I + h(t
1
t
0
) +h(t
1
t
0
)
= I + h(t
1
t
0
) I using (a)
114
h is same as in the actual path and varied path h = 0
A = 0
The P.L.A. A =
t
1
_
t
0
n

i=1
p
i
q
i
dt = 0
Hence the theorem.
Derive Hamilton principle form of least action
Solution :
Write down upto form the above.
Let us assume that the variation be contemperaneous. In this case
t = 0.
Second integral vanished.
Let us assume all the applied force are derivable from a potential
function.
Hence the least integral vanishes.
Let us assume all the varied paths have xed end points in a q
space.
Hence the I, integral vanishes.
I = 0
This is Hamilton principle.
Obtain Jacobis form of principle of least action
Solution :
Write the bookwork Least action completely let us assume that the
system be natural.
Now
n

i=1
p
i
q
i
L = H
n

i=1
p
i
q
i
= L + H
= T
2
T
0
+ V + T V
115
= T
2
T
0
+ T
= T
2
T
0
+ T
2
+ T
0
+ T
1
= 2T
2
+ T
1
[ T + v = h]
= 2T + 0
n

i=1
p
i
q
i
= 2T
For the natural system T
2
= T, T
1
= 0, T
0
= 0.
Least action = A =
t
1
_
t
0
n

i=1
p
i
q
i
dt = 0
A =
t
1
_
t
0
2Tdt = 0
A =
t
1
_
t
0
2Tdt = 0
Dene ds as follows
ds
2
=

j
m
ij
q
i
q
j
dt
2
= 2Tdt
2
ds =

2Tdt
A =
t
1
_
t
0

2T

2Tdt
=
t
1
_
t
0

2Tds
=
t
1
_
t
0
_
2 (h v)ds = 0
This is the principle of least action in Jacobis theorem.
116
Problem :
Given a mass spring system consisting of a mass m and a linear
spring of stiness of k as shown in gigure. Find the equation of motion
using the Hamiltons procedure. Assume that the displacement x is
measure from the unstressed position of the spring.
Solution :
First we nd the K.E and P.E in the usual form for mass spring
system we obtain
T =
1
2
m x
2
V =
1
2
x
2
k
where k is the stiness.
But we know that
L = T V =
1
2
m x
2

1
2
x
2
k
Angular momentum P =
L
x
=
1
2
m2 x
= m x[ here x is only the co-ordinates q = x]
P = m x
x =
P
m
T =
1
2
m x
2
=
P
2
2m
The Hamilton function is
H (x, p) = p x L
= p (pm)
_
1
2
m(pm)
2

1
2
kx
2
_
=
p
2
m

p
2
2m
+
1
2
kx
2
117
=
p
2
m
+
1
2
kx
2
(1)
H (q, p, t) = H (x, p)
Using Hamilton canonical equation
q
i
=
H
p
; x =
H
p
=
2
2m
p =
p
m
p =
H
x
=
_
1
2
k 2x
_
= kx
Using (1)
p = kx (2)
But
p = m x
p = m x (3)
From (2) and (3)
m x = kx
mx + kx = 0
This is equation of the mass spring system.
Keplers problem using Hamilton canonical function.
A particle of mass m is attached to a xed point O by an inverse
square force that is F
r
=
m
r
2
where is the gravitational co-ecient
using the plane co-ordinates (r, ) to describe the position of the par-
ticle.
Find the equation of motion
q
i
=
H
p
i
; p
i
=
H
q
i
.
118
Solution:
T =
1
2
m
_
r
2
+ r
2

2
_
V =
_
F
r
dr =
_

m
r
2
dr
The Lagranges function
L = T V
H = T + V
=
1
2
m
_
r
2
+ r
2

2
_

m
r
=
1
2
m
_
p
2
r
m
2
+ r
2
p
2

m
2
r
4
_

m
r
=
p
2
r
2m
+
p
2

2mr
2

m
r
Using the Hamilton equation we have
r =
H
p
r
=
2p
r
2m
=
p
r
m
(1)
p
r
=
H
r
=
_
1
2m
p
2

(2) r
3
+
m
r
2
_
=
p
2

mr
3

m
r
2
(2)
From (1)
r =
p
r
m
r =
p r
m
p

=
H

r
1
m
_
p
2

mr
3
_
+
1
m
_
m
r
2
_
= 0
m r
p
2

mr
3
+
m
r
2
= 0
119
p

is constant say
m r

2
mr
3
+
m
r
2
= 0
p
r
= m r
p
r
= m r (3)
From equation (1) and (2)
m r =

2
mr
3

m
r
2
m r

2
mr
3
+
m
r
2
= 0
This is the equation of motion.
Problem
Using the Hamilton equation discuss the motion for a charged par-
ticle in an electro magnetic eld.
Solution :
The Lagrangian function is
T =
1
2
mv
2
V =
_

v

A
_
L = T V
=
1
2
mv
2

v

A
_
Momentum
p = m v +

A
m v = p

A
v =
p

A
m
Assume is the charge our the particle Hamilton H is given by
H = p v L
120
= p v
1
2
mv
2
+
_

v

A
_
=
_
m v +

A
_
v
1
2
m v
2
+

v

A
= m v
2
+

A v
1
2
m v
2
+

v

A
=
1
2
m v
2
+

=
1
2
m
_
p A
m
_
2
+

=
1
2m
(p A)
2
+

=
1
2m
_
(p
x
A
x
)
2
+ (p
y
A
y
)
2
+ (p
z
A
z
)
2

x =
H
p
x
=
1
2m
[2 (p
x
A
x
)]
=
1
m
(p
x
A
x
)
Similarly
y =
1
m
(p
y
A
y
)
z =
1
m
(p
z
A
z
)
v =
1
m
_
p

A
_
The second canonical equation
x =
H
p
x
p
x
=
H
x
=
1
2m
_
2 (p
x
A
x
)
_

A
x
x
_
+ 2 (p
y
A
y
)
_

A
y
x
_
+2 (p
z
A
z
)
_

A
z
x
__
+

x
121
=

x
+

m
_
(p
x
A
x
)
A
x
x
+ (p
y
A
y
)
A
y
x
+(p
z
A
z
)
A
z
x
_
(1)
Similarly
p
y
=

y
+

m
_
(p
x
A
x
)
A
x
y
+ (p
y
A
y
)
A
y
y
+ (p
z
A
z
)
A
z
y
_
p
z
=

z
+

m
_
(p
x
A
x
)
A
x
z
+ (p
y
A
y
)
A
y
z
+ (p
z
A
z
)
A
z
z
_
But

_
v

A
_
= [( x
i
+ y
j
+ z
k
) (Ax
i
+ Ay
j
+ Az
k
)]
= ( xA
x
+ yA
y
+ zA
z
)
=

i
_
x

x
A
x
+ y

y
A
y
+ z

z
A
z
_
Substitute the values of x, y , z, we get

_
v

A
_
=

1
m
i
_
(p
x
A
x
)
A
x
x
+ (p
y
A
y
)
A
y
x
+(p
z
A
z
)
A
z
x
_
=

i
_
p
x

+

x
_
=
_
p
x

+

x
_
i +
_
p
y

+

y
_
j +
_
p
z

+

z
_
k
=
p

+
p =
_
v

A
_

Hence the proof.


Lagenders Transformation :
122
Derive Hamiltons cononical equation using Legender transforma-
tion and Lagranges equation.
Solution : Suppose the funcion F (u
1
, u
2
, , u
n
,
1
,
2
, ,
m
, t)
is associated with the given system.
Hence us are active variables and s and t are passive variable.
Let
v
i
=
F
u
i
(i = 1, 2, , n) . (1)
Let nm Heissian determinant for the transformation be non-zero.
Then

2
f
u
i
u
j

u
j
_
F
u
i
_

v
i
u
j

= 0.
Now let us dene a new function G(v
1
, v
2
, ., v
n
,
1
,
2
, ,
n
, t)
according to the equation
G =
n

i=1
u
i
v
i
f (u, , t) . (2)
Consider a variation G associated with arbitrary variation of the
active variable.So
G =
n

i=1
G
v
i
v
i
=
n

i=1
_
u
i
v
i
+ v
i
u
i

F
u
i
u
i
_
or
n

i=1
G
v
i
v
i
=
n

i=1
_
u
i
v
i
+
_
v
i
u
i

F
u
i
_
u
i
_
where us and vs are assumed to the arbitrary equation .
Equating the co-ecient on both sides
u
i
=
G
v
i
. (3)
For a holonomic system
123
The Lagranges equation is
p
i
=
L
q
i
, i = 1, 2, 3, , n (4)
where p
i
=
L
q
i
, i = 1, 2, 3, , n.
Let us choose L(q, q, t) corresponding to the function F (u, , t) .
Also we have

2
L
q
i
q
j

= |m
ij
| = 0.
Since the inertia matrix is positive denite matrix.
Let G(u, , t) corresponding to the Hamiltons funciton
H (p, q, t) =
n

i=1
p
i
q
i
L
where q
i
=
H
p
i
.

G
v
i
= u
i
H
q
i
(q, p, t) =
L
q
i
(q, q, t) (5)
From (4) and (5)
H
q
i
= p
i
p
i
=
H
q
i
Problem:
Obtain Hamiton cononical eqn from modied Hamiltons principle.
Solution :
Consider a holonomic system having n independent qs.
By Hamiltons principle, I = 0 where I =
t
1
_
t
0
Ldt.
I =
t
1
_
t
0
Ldt
124
we have
h = T + V, L = T V
H =

p
i
q
i
L
L =

p
i
q
i
H (p, q, t)

t
1
_
t
0
Ldt =
t
1
_
t
0
_

p
i
q
i
H (p, q, t)
_
dt
0 =
t
1
_
t
0
_
n

i=1
(p
i
q
i
+ q
i
p
i
)
n

i=1
_
H
p
i
p
i
+
H
q
i
q
i

H
t
t
_
_
dt
125
UNIT IV
Hamilton -Jacobi theorem
Integrals of Motion :
In a holonomic system consisting of n generalized co-ordinates.
We have n second order non-linear dierential equation with time as
independent variables. Any general analytic solution of this dierential
equation contains one method of expression the general solution.
ie, To obtain R independent funciton of the form
f
i
(q, q, t) =
j
, j = 1, 2, 3, , 2n
We have the
j
s are arbitrary constant.
These two functions are called the integral.
The value of each s are depending on the initial condition. In
principle these 2n equations can be solved for the qs as function of
and t.
It is possible to nd qs and qs as a function of and t.
It is possible to nd
q
i
= q
i
(
1
,
2
, ,
2n
, t)
q
i
= q
i
(
1
,
2
, ,
2n
, t)
1. Corresponding Hamiltons Principle function. Solve the Hamil-
ton poblem giving the motion in phase space as function of time.
Solution :
Let
I =
t
1
_
t
0
Ldt.
Consider the cononical integral which is associating with Hamil-
tons principle.
126
We want to evaluate this integral over actual dynamical path of a
holonomic system that obeys standard Lagranges equation or Hamil-
tons equation. If 2n independent initial conditions as specied at the
time t
0
any nal time t
1
.
Let us consider the solution of equations
q
i1
= q
i1
(q
0
, q
0
, t
0
, t
1
) , i = 1, 2, 3, , n.
Let us assume Jacobian
(q
11
, q
12
, , q
1n
)
( q
10
, q
20
, , q
n0
)
is non zero. Hence we get
q
i0
=
i
(q
0
, q
1
, t
0
, t
1
) , i = 1, 2, 3, , n
We can evaluate the integral
t
1
_
t
0
Ldt as a funciton of (q
0
, q
0
, t
0
, t
1
) .
Then subsititution for q
i0
from the above equation we have
s = s (q
0
, q
1
, t
0
, t
1
) =
t
1
_
t
0
Ldt.
The function s (q
0
, q
1
, t
0
, t
1
) obtain from the canonical equation.
Required form to be obtained.
Let us assume twice dierentiation in all its arguments. This is
called Hamiltons principle function.
Let us consider a non contemporaneous system general variation
of the canonical integral with the reference in the actual solution.
We have,
H H (q
1
, p, t)
dH
dt
=
n

i=1
_
H
q
i
dq
i
dt
+
H
p
i
dp
i
dt
_
+
H
t
dt
127
=
n

i=1
_
H
q
i
q
i
+
H
p
i
p
i
_
+
H
t
(1)
From Hamiltons canonical equation we have
q
i
=
H
p
i
, p
i
=
H
q
i
, i = 1, 2, , n
L
t
=
H
t
(1)

H =
H
t
=
L
t
We know that from the principle of least action
I =
t
1
_
t
0
n

i=1
d
dt
_
L
q
i
_
q
i
dt +
t
1
_
t
0
L
t
t
__
n

i=1
L
q
i
q
i
L
_
d
dt
_
dt

t
1
_
t
0
_
n

i=1
d
dt
_
L
q
i
_

L
q
i
_
q
i
dt
=
t
1
_
t
0
n

i=1
d
dt
(p
i
q
i
) dt +
t
1
_
t
0
_

H
t
t H
d
dt
(t)
_
dt
=
t
1
_
t
0
n

i=1
d
dt
(p
i
q
i
) dt
t
1
_
t
0
d
dt
(Ht) dt
As our assumtion S = I.
S =
_
n

i=1
p
i
q
i
Ht
_
t
1
t
0
dS =
n

i=1
p
i
q
i

t
1
_
t
0
d
dt
(Ht) dt.
This is dierence of two forces.
(By writting the above quation in dierential form)
s = s (q
0
, q
1
, t
0
, t
1
)
128
ds =
n

i=1
_
s
q
i1
q
i1
+
s
q
i0
q
i0
_
+
s
t
1
t
1
+
s
t
0
t
0
(2)
The (2n+2) arguments of the principle funcitons are independent.
Hence the corresponding co-ecients in (1) and (2) are equal.
p
i1
=
s
q
i1
, p
i0
=
s
q
i0
, i = 1, 2, 3, ., n
H
1
=
s
t
1
; H
0
=
s
t
0
Now, we get p
i0
as function f (q
i0
, q
i1
, t
0
, t
1
) .
Assuning

2
s
q
i0
q
j1

= 0
We can solve for each q
i1
as a funciton of the initial condition
i.e., q
i1
= q
i1
(q
0
, p
0
, t
0
, t
1
) , i = 1, 2, 3, , n.
If this is substituted in p
i1
=
s
q
i1
, we have
p
i1
= p
i1
(q
0
, p
0
, t
0
, t
1
) .
This completes the solution of the Hamiltons problem. Note :
(i)
t
1
_
t
0
Ldt is a fucntion (q
i1
, q
i1
, t
0
, t
1
) but q
i1
, q
i1
are the funciton of
q
0
, q
0
, t
0
, t
1
,
(ii) In the whole discussion rst nding the principle funcion without
knowing of solution.
Pfaan Dierential forms :
i) A pfaan form g is the m variables x
1
, x
2
, , x
m
can be writ-
ten as
= X
1
(x) dx
1
+ + X
m
(x) dx
m.
129
Let us dene C
ij
=
X
i
x
j

X
j
x
i
ii) If the pfaan form is an exact dierentiation then all the ds are
zero
we have
ds =
n

i=1
p
i1
dq
i1

i=1
p
i0
dq
i0
H
1
dt
1
H
0
dt
0
.
The R.H.S consists of twon variables are ps and qs each expression
can be written as
p
1
q
1
+ p
2
q
2
+ + p
n
q
n
+ 0 dp
1
+ + 0 dp
n
1 + h(q, p
t
) dt
Now, we have m = 2n + 1 (odd) .
By pfacan system these equation are of the form
n

i=1
C
ij
dx
i
= 0, j = 1, 2, 3, , m.
Applying this equation to the dierential equaiton,we have
dq
j

H
p
j
dt = 0 (1)
dp
j

H
q
j
dt = 0 (2)
(1) and (2) can be written as
(1) q
j
=
H
p
j
(2) p
j
=
H
q
j
, j = 1, 2, 3, , n
H = H (q, p, t)
H
t
=
n

i=1
_
H
q
i
q
i
+
H
p
i
p
i
_
+
H
t
These are the Hamiltons canonical equation.
130
Also using (2), we have

H =
H
t
.
Now let us generalized the dierential form
ds =
n

i=1
p
i1
dq
i1

i=1
p
i0
dq
i0
H
1
dt
1
+ H
0
dt
0
.
Using the 2n parameter (
1
,
2
, ,
2n
) to specify the intial con-
dition in place the q
0
s and p
0
s in otherwise.
We assume a transformation
q
i0
= q
i0
(
11
,
21
, ,
2n
)
p
i0
= p
i0
(
11
,
21
, ,
2n
)
_
(3)
such that
(q
10
, q
20
, , q
n0
, p
10
, p
20
, , p
n0
)
(
11
,
21
, ,
2n
)
= 0
Then using (3) we have
n

i=1
p
i0
dq
i0
=
2n

i=1
p
i0
q
i0

j
(4)
By the theorem that the 2n can be replaced by ns and ns where
the function

i
=
i
(
11
,
21
, ,
2n
)

i
=
i
(
11
,
21
, ,
2n
) , i = 1, 2, 3, , n.
choosen such that
n

i=1

i
d
i
=
2n

i=1
(, d)
131
in the same manner as (4) we have

j
() =
n

i=1

j
, j = 1, 2, 3, , n.
It can be show that s are not unique
n

i=1
p
i0
dq
i0
=
n

i=1

j
d
i
where s and s are another representation shows that (q
0
, p
0
) and
(, ) are connected by a homogeneouss cannonical transformation at
a time t
0
.
Hamilton - Jacobi equation :
Consider a holonomic system giving 2n independent initial condi-
tions at time to as q
0
= p
0
.
Now we have dierential equation
Assume that
ds =
n

i=1
p
i1
dq
i1

i=1
p
i0
dq
i0
H
1
dt
1
+ H
0
dt
0
(1)
where s is the Hamiltons principle funtion. It is associated with
cononical transformation relating the initial and nal point of a path
in a phase space.
Let the initial condition be specied by

i
=
i
(q
10
, q
20
, , q
n0
, p
10
, p
20
, , p
n0
)

i
=
i
(q
10
, q
20
, , q
n0
, p
10
, p
20
, , p
n0
)
which satised.
n

i=1
p
i0
dq
i0
=
n

i=1

i
d
i
ds =
n

i=1
p
i1
dq
i1

i=1

i
d
i
H
1
dt
1
+ H
0
dt
0
(2)
132
Now we consider s number can be associated as a function of
s = s (q
i1
,
i
, t
1
, t
0
)
ds =
n

i=1
ds
dq
i1
dq
i1
+
n

i=1
s

i
d
i
+
s
t
1
dt
1
+
s
t
0
dt
0
(3)
Hence qs and s are independent equality variable.
Equating the corresponding coecient in ds we have

i
=
s

i
, i = 1, 2, 3, , n.
p
i1
=
s
q
i1
, H
1
=
s
t
1
, H
0
=
s
t
0
.
Let us assume initial time t
0
= 0 , take the nal time at t.
i.e., t
1
= t dt
1
= dt
Hence we can drop the subsorip qs . Hence (1) becomes
ds =
n

i=1
p
i
dq
i

i=1

i
d
i
Hdt. (I)
From this we see that s is a funciton of (q, , t)
(p is a function of q, q, r, t )
ds =
n

i=1
s
q
i
dq
i
+
n

i=1
s

i
d
i
+
s
t
dt (II)
Once again assuming

2
s
q
i

= 0.
Equating the coecient in I and II, we have

i
=
s

i
, i = 1, 2, 3, , n (4)
p
i
=
s
q
i
, i = 1, 2, 3, , n (5)
133
s
t
= H (6)
Form (4) we can get qs as function (, , t) using this in equation (5)
we can nd p as function of (, , t) .
Hence we have the solution for Hamiltons rpinciple.
H is usually as a function of (q, p, t) . Using (5) and (6) we have
s
t
+ H (q, p, t) = 0
s
t
+ H
_
q,
s
q
, t
_
= 0
This is called Hamilton - Jacobi equaiton.
State and prove Jacobi theorem.
Statement :
If s (q, , t) is any complete solution of the Hamilton Jacobi equa-
tion
s
t
+ H
_
q,
s
q
, t
_
= 0

i
=
s

i
, i = 1, 2, 3, , n (1)
p
i
=
s
q
i
, i = 1, 2, 3, , n
where s are arbitrary constant are used to solve for q
i
(, , t) and
p
i
(, , t) .
Then these expression provide the general solution of the canonical
equation associated with Hamilton H (q, p, t) .
OR
Prove that any complete solution of the Hamilton Jacobi
equation leads to a solution of this Hamilton problem.
Proof :
s
t
+ H
_
q,
s
q
, t
_
= 0 (2)
s
q
i
= p
i
, i = 1, 2, 3, , n (3)
134
which is a function of (q, , t) . Dierentiating w.r.to
i
d
2
s
d
i
t
+
n

j=1
H
p
j
p
j

i
= 0 (4)
where p
j
is considered as a function of (q, , t) in (3).
[p (q, x, t) are independent to each other
ds
dq
i
= p
i
]
s

i
is a function of (q, , t) and s and s are constants.
Now,
s

i
=
i
, i = 1, 2, 3, , n
Taking the total time derivation of this w.r.to t

2
s
t
i
+
n

j=1

2
s
q
j

i
q
j
= 0 (5)

2
s
t
i
+
n

j=1

q
j
_
q
j
t
__
s

i
_
dt = 0
(5) and (4) Using (1),(2) and (3), we have
n

j=1
_
q
j

H
q
j

i
_

2
s
q
j

i
= 0, i = 1, 2, 3, , n (6)
But

2
s
q
i

= 0.
q
j
=
H
p
j
, j = 1, 2, 3, , n (7)
Now,
s
t
+H (q, p, t) = 0.p
i
is a function of (q, , t), dierentiating (1)
partially w.r.to q
j
, we have

2
s
q
j
t
+
n

i=1
H
p
i
p
i
q
j
+
H
q
j
= 0 (8)
135
(3) p
j
=
s (q, , t)
q
j
, j = 1, 2, 3, ..., n.
Taking the total time derivation of this equation we have to dieren-
tiating w.r.to t
p
j


2
s
tq
j

i=1

2
s
q
j
q
i
q
i
= 0 (9)
(6) + (7)
p
j
+
H
q
j
+
n

i=1
H
p
i
p
i
q
j

i=1

2
s
q
j
q
i
q
i
= 0
p
j
+
H
q
j
+
n

i=1
q
i
p
i
q
j

i=1
p
i
q
j
q
i
= 0
p
j
+
H
q
j
= 0, p
i
=
s
q
i
, j = 1, 2, 3, ..., n
p
j
=
H
q
j
Equations (7) and (10) together give the cononical equation of Hamil-
ton. Thus any complete solution of the Hamilton Jacobi equation leads
to a solution of the Hamilton equation.
Modied Hamilton Jacobi equation :
Consider a conservative holonomic system with n independent for
this system H is not an explicit function of time t for this system
H (q, p) =
n
= h where h is the value of the Hamilton Jacobi integral.
s
t
+ H
_
q,
s
q
, t
_
= 0
s
t
+ H (q, p) = 0
s
t
= H =
n
(1)
136
s can be taken as linera funciton of time t is
s (q, , t) =
n
t + (q, ) .
Then we have omitted arbitrary additive constant.
The funciton
(q, ) = (q
1
, q
2
, ..., q
n
,
1
,
2
, ...,
n
) .
does not contain time explicitly and is known as a characteristic func-
tion.
Now,
s

i
=

i
, i = 1, 2, 3, ..., (n 1) (2)
s

n
=

n
t (3)
s
q
i
=

q
i
, i = 1, 2, 3, ..., n (4)
Using (3) and (4), Hamilton jacobi equaiton Induces to
This is the modied Hamilton Jacobi equaiton
Note:
We have

i
=
s

i
, p
i
=
s
q
i
s

i
=

i
, i = 1, 2, 3, ..., (n 1)
s

n
=

n
t
i
s
q
i
=

q
i
, i = 1, 2, 3, ..., n
Comparing the equation we have

i
=

q
i
, i = 1, 2, 3, ..., n
137
where
n
is the intial time to t
0
.
Ignorable co-ordinates :
Consider a holonomic system described by the standard form
d
dt
_
L
q
i
_

L
q
i
= 0,
q
i
is said to be an ignorable co-ordinate if
L
q
i
= 0.
Also p
i
=
L
q
i
=
i
only for the ignorable co-ordinate.
Case (i)
Consider a system with ignorable co-ordinates q
1
, q
2
, ..., q
n
.Let us
assume the system is not consecutive. We know that
p
i
=
i
, i = 1, 2, 3, ..., k.
We can assume principle solution is of the form
s (q, , t) =
k

i=1

i
q
i
+ s

(q
k+1
, ..., q
n
,
1
,
2
, ...,
n
, t)
Hamilton Jacobi equation leads to
s
t
+ H
_
q,
s
q
, t
_
= 0
s

t
+ H
_
q
k+1
, ..., q
n
,
1
,
2
, ...,
k
,
s

k+1
, ...,
s

n
, t
_
= 0.
The complete solution of these equation involve (n k) non additive
constants exclusive of the known solution of this equation is obtained
form.

i
= q
i
+
s

i
, i = 1, 2, 3, ..., k
p
i
=
i
, i = 1, 2, 3, ..., k
p
i
=
s

q
i
, i = k + 1, k + 2, k + 3, ..., n
p
i
= q
i0
138

i
=
s

i
, i = k + 1, k + 2, k + 3, ..., n
Case (ii)
Consider a system with ignorable co-ordinate q
1
, q
2
, ..., , q
k
and con-
servative with the help of the previous result we have
s (q, , t) =
k

i=1

i
q
i

n
t +

(q
k+1
, ..., q
n
,
1
,
2
,
n
, t)
The modied Hamilton Jacobi equation becomes
H
_
q
k+1
, ..., q
n
,
1
,
2
, ...,
k
,

k+1
, ...,

n
_
= q
n
The complete solution for

in this case involve (n k 1) non addi-


tive constants
k+1
,
k+2
, ...,
n1
the energy constent
n
and constant
moments
1
,
2
, ...,
k
.
The motion of the system is given by

i
= q
i
+

i
, i = 1, 2, 3, ..., k

i
=

i
, i = 1, 2, 3, ..., (k 1)
t
n
=

n
p
i
=
i
(i = 1, 2, 3, ..., k) p
i
=

q
i
, (i = k + 1, k + 2, ..., n)
Ordinary Conservation of linear momentum
This follows from Newtons second law of motion
d
dt
(mv) =
dp
dt
= F.
If the total force is zero, then
dp
dt
= 0 at the linear momentum is
conserved.
Note :
The word conservation applied in the of connectedness.
139
Conservation of angular momentum :
Angular momentum is the angular of linear momentum in the case
of relational motion.
Consider a particle at mass m and the linear momentum p at a
positin vector r relative to the origin O at an inertial frame.
Angular momentum l at the particle with respect to the origin O
is L = r
Moment of a force about a origin is N = r F then F =
dp
dt
.
N = r F N = r
dp
dt
(1)
N =
d
dt
(r
i
)
dr
dt

N =
d
dt
(r ) v mv (2)
N =
d
dt
(r ) 0 (3)
N =
d
dt
(r ) (4)
The second term is zero both vectors are parallel now.
N =
d
dt
(r )
=
dL
dt
.
Thus the rate of change of angular momentum of a particle is equal
to the force acting on it is the total force N = 0.
Then
dL
dt
= 0,angular momentum is considered in the absence of
an external force.
Principle of conservation of evergy :
Let the only force acting on the given particle be conservation is
of the force are derivable from scalar potential evergy function.
F = V, then the T.E.=K.E.+P.E.
Suppose under the action of such force F particles moves from the
140
position 1 to 2. Then the workdone

2
=
2
_
1
Fds
=
_
Fds =
2
_
1
Fds =
2
_
1
dF
dt
ds
The particle moves ds distance in the time dt with velocity r, so
p = mv = m r.
dr
dt
= r, dr = rdt

2
=
2
_
1
d
dt
(m r) rdt
=
2
_
1
d
dt
_
m r
2
_
dt
=
2
_
1
2
d
dt
_
1
2
m r
2
_
dt
= T
2
T
1
T
2
, T
1
denote the K.E of the particle of the K.E of the position 2 and
1 respectively.
F = V

12
=
2
_
1
V dr =
2
_
1

dV
dx
dr

2
_
1
dV = V
1
V
2
141
From (1) and (2),
T
2
T
1
= V
1
V
2
T
2
+ V
2
= V
1
+ T
1
= constant
T + V = constant
T E of a particle is conserved (constant)
Problem :
For a simple spring system using Hamilton Jacobi method to solve
it.
The K.E and P.E are given by
T =
1
2
m x
2
, V =
1
2
kx
2
P =
T
x
=
1
2
m2 x = m x
x =

m
x
2
=
P
2
m
2
H = T + V
=
P
2
2m
+
kx
2
2
We are considering a conservative systm.
H = T + V =
The modied homilton Jacobi equation is
1
2m
_

x
_
2
+
1
2
kx
2
=
_

x
_
2
= 2m
_

1
2
kx
2
_
142
where is energy constant.

x
=

2m
_

1
2
kx
2
_
=

2m
_
2 kx
2
2
_
=

mk

_
2
k
x
2
_
Taking
a =
_
2
m
2
and =
_
k
m

x
=
x
_
x
0

mxdx
x
_
x
0
dt
_
a
2

2
(x, ) = m
x
_
x
0
_
a
2

2
dt, =
_
x
m

= m
_
1
2
2a

a
2

2
1
2
_
2

2
m
2
m
2
d
= t
=
1

x
_
x
0
dt

a
2
t
2
=
1

x
_
x
0
dt

t
2
a
2
=
1

_
cos
1
t
a
_
x
1
x
0
=
1

_
cos
1
x
0
a
cos
1
x
1
a
_
is initial time t
0
. i.e., = t
0
.
t t
0
=
1

_
cos
1
_
x
a
__
(t t
0
) = cos
1
_
x
a
_
143
(t t
0
) = cos
1
_
x
a
_
[ (t t
0
) ] = cos
1
_
x
a
_
x = a cos [ (t t
0
) ]
=
_
2
m
2
cos [ (t t
0
) ] .
Let x(t
0
) = x
0
. Then x (t
0
) = v
0
. We know that
= T + V
=
1
2
m x
2
+
1
2
kx
2
=
1
2
mv
2
0
+
1
2
k
2
x
2
0
=
m
2
2
_
v
2
0

2
+ x
2
0
_
(1)
sin =
_
1 cos
2

=
_
a
2
x
2
0
a
2
=
1
a
_
a
2
x
2
0
=
1
a
_
2
m
2
x
2
0
using (1)
=
1
a
_
v
2
0

2
+ x
2
0
x
2
0
=
v
0
a
Now,
x = a cos [ (t t
0
) ]
= a [cos (t t
0
) cos + sin (t t
0
) sin ]
=
x
0
cos
[cos (t t
0
) cos + sin (t t
0
) sin ]
= x
0
cos (t t
0
) +
x
0
cos
sin (t t
0
)
v
0
a
= x
0
cos (t t
0
) + a sin (t t
0
)
v
0
a
144
= x
0
cos (t t
0
) +
v
0

sin (t t
0
)
Dierentiating,
x = 0
Amplitute oscillation is given by
a =
_
v
2
0

2
+ x
2
0
Keplers Problem :
Use Hamilton Jacoi equation to analysis the keplers problem or
modied Jacobi method.
Solution :
Suppose a particle of unit mass attacted by an inverse square grav-
itational force at a xed point O. The position of a given problem
is given interms of the polar co-ordinates (r, ) in the plane of the
orbits.
The K.E and P.E. are
T =
1
2
m
_
r
2
+ r
2

2
_
V =

r
where is gravitation coecient.
Lagrangian function is
L = T V
=
1
2
m
_
r
2
+ r
2

2
_
+

r
We nd that the generalized momentum are given by
p
r
=
L
r
;

=
p

r
2

= p

=
L

= r
2

145

r
2
The system is natural.
H = T + V
H =
1
2
_
p
2
r
+
p

r
2
_


r
=
t
(1)
where
t
represents constants values of the total energy (1) does not
appear in H.L.
It is ignorable.
L

= 0
L

= p
0
= constant =
0
The modied Hamilton Jacobi equation is
1
2
_

r
_
2
+

2

2r
2


r
=
t
r is ignorable.
1
2
_

r
_
2
=
t
+

r


2

2r
2
_

r
_
2
= 2
_

t
+

r


2

2r
2
_

r
=
_
2
t
+ 2

r


2

r
2

=
r
_
r
0
_
2
t
+ 2

r


2

r
2
dr
We have

i
= q
i
+


0
=

146
=
r
_
r
0
_

1
r
2
_
2

2
_
2
t
+ 2

r


2

r
2
dr
=
r
_
r
0

r
2
_
2
t
+ 2

r


2

r
2
dr
=
r
_
r
0
dr
r
2

_
2
t

+ 2

r

1
r
2
=
r
_
r
0
dr
r
2
_
2
t

+
2
r

1
r
2
=
r
_
r
0
dr
r
2

2
t

+

2

_
1
r

_
2
=
r
_
r
0
d
_
1
r

_
r
2

2
t
+
2

_
1
r

_
2

0
= cos
1
_

_
1
r

_
2
t

+
2

_
r
r
0
Let =
0
where r = r
0
.

0
= cos
1
_

2

r
r
_
2
t
+
2

+
2
_
cos
1
_
r
0
r
o
_

2
_
= cos
1
_

2

r
r
_
2
t
+
2

+
2
_
cos
1
(1)
cos =
r +
2

r
_
2
t
+
2

+
2
147

0
= 0 where r
0
= 0
_
2
t

+ 1 cos =
r
+
2

2
t

+ 1

2
cos =
+

2

r
= 1 +

1 + 2
t

2
cos
This is the form
l
r
= 1 + e cos .
This the cone whose eccentricity is
e =

2
+ 2
t

2
This is the equation of a conic section having eccentricity.
r =

2

_
_
1 +

2
+ 2
t

2
cos
_
_
Hence the Keplers problem.
Seperability :
The index of seperability associated with the solution of P.D.E by
a solution to that is by expressing the solution interms of integrals
each involving only one variable.
Orthogonal system :
It is conservative holonomic system whose K.E function contains
only squared forms and no produed tems in this variable. If a system
is seperable that is possible to nd a charatorstic function such that
=
n

i=1

i
(q
i
) where each contain only one of the qs.
Note :
A particularly simple example of a seperable system occure in the
all but of the co-ordinates all ignorable.
148
Liouvilles System :
It is an orthogonal system whose K.E. and P.E. are of the form
T =
1
2
_
r

i=1
p
i
(q
i
)
_
n

i=r
_
( q
i
)
2
R
i
(q
i
)
_
(1)
=
R
1
P
2
1
+ R
2
P
2
2
+ ... + R
n
P
2
n
2 (f
1
+ f
2
+ ... + f
n
)
=
v
1
(q
1
) + v
2
(q
2
) + ... + v
n
(q
n
)
f
1
(q
1
) + f
2
(q
2
) + ... + f
n
(q
n
)
where f
i
, q
i
and v
i
are each function of q
i
. We assume that
n

i=1
f
i
(q
i
) > 0
and R
i
(q
i
) > 0.
1
2
a =
2

a = 2
2
2
This is identical with (a).
Hence the system is seperable.
Using the pfaan equation, derive the Hamilton canonical equation
from Hamilton funciton
ds =
n

i=1
p
i1
dq
i1

i=1
p
i0
dq
i0
H
1
dt
1
+ H
0
dt
0
Solution :
Book Work 5: p equation.
Discuss the Keplers probelm using seperability.
Solution :
Let us consider a particle of unit mass p attached towards O by
an inverse square gravitational force. Let T be the K,E, of the system.
Let V be the P.E. of the system.
T =
1
2
_
r
2
+ r
2

2
+ r
2

2
sin
2

_
dr =

r
149
L = T V =
1
2
_
r
2
+ r
2

2
+ r
2

2
sin
2

_
+

r
Now
p
r
=
L
r
=
1
2
2 r = r
p

=
L

=
1
2
r
2
2

= r
2

=
L

=
1
2
r
2
sin
2
2

= r
2
sin
2

T =
1
2
_
p
2
r
+
p
2

r
2
+
p
2

r
2
sin
2

_
where p
r
, p

and p

are the generalized moments. This system is an


orthogonal system.
we have H = T + V =
t
.
The modied Hamilton Jacobi equation is
1
2
_

r
r
_
2
+
1
2r
2
_

_
2
+
1
2r
2
sin
2

_
1


r
=
t
where
=
r
(r) +

() +

() (1)
is missing from the above equation since is ignorable co-
ordinates.
[r

() =

()] p

(1) 2r
2
r
2
_

r
r
_
2
2r
2
_

r
+
t
_
+
_

_
2
+
1
sin
2

)
2
= 0 (2)
The rst two terms are functions of r only and the last terms are
fuctions of .
Hence they are each to seperable constant.
Stackels thoorem
150
Statement : Consider an orthogonal system whose K.E is given by
T =
1
2
n

i=1
m
i
q
2
i
=
1
2
n

i=1
c
i
p
2
i
where c
i
(q
1
, q
2
, ..., q
n
) > 0. Stackels theorem asserts that this is a
seperable system i there exists a non-singular matrix n n[
ij
(q
i
)]
and a column matrix [(q
i
)] exists such that
i) C
T
= (1, 0, ..., 0)
ii) C
T
= V, where V (q
1
, q
2
, ..., q
n
) is the P.E. and C is a column
matrix composed of the ncs.
Proof :
Necesary part :
Let us assume that the general orthogonal system be seperable.
Hence it has a characterstic function (q, ) which consist of the
sum of terms of the form
i
(q
i
,
1
,
2
, ...,
n
) .
This characteristic function is the complete integral of the modied
H.J equation
1
2
n

i=1
c
i
_

i
q
i
_
2
+ V =
1
(1)
Here
1
is chosen as the total energy because of the system is assumed
to be separability.

i
q
i
_
2
is a function of (q
i
,
1
,
2
, ...,
n
) .
We can choose the seperation constants such that the ds appear
linearly.
The most general form involving the single co-ordinate q
i
is
_

i
q
i
_
2
= 2
i
(q
i
) + 2
n

j=1

ij
(q
j
)
j
(I)
where the mumerical co-ecients are chosen for our convenient for our
convenient
151
Then subustitute (I) in (1)
1
2
n

i=1
c
i
_
2
i
(q
i
) + 2
n

j=1

ij
(q
j
)
j
_
+ V =
1
(2)

i=1
c
i

i
(q
i
) +
n

i=1
n

j=1
c
i

ij
(q
j
)
j
+ V =
1
C
T
+ C
T

+ V =
1
.
Comparing the terms containing s we nd that C
T
= (1, 0, 0, ..., 0) ,which
we recognise as the rst Stackels conditions. Similary the term in (3)
which do not involve s must sum to zero reading to C
T
= V,which
is Stackels condition.
Sucient part :
Dene a column matrix a =
_

i
q
i
_
2
.
The modied Hamilton Jacobi equation (1) can be written as
C
T
(a) + V = .
Using second Stacicels condition
1
2
C
T
a + C
T
= (1, 0, 0, ..., 0) (4)
C
T
= (1, 0, 0, ..., 0)
_
C
T
= V
_
C
T
= (1, 0, 0, ..., 0)
1
(5)
Sustituting (5) in (4)
1
2
(1, 0, 0, ..., 0) + C
T
=
1
1
2
(1, 0, 0, ..., 0)
1
a + (1, 0, 0, ..., 0)
1
= (1, 0, 0, ..., 0)
152
(1, 0, 0, ..., 0)
_
1
2

1
a +
1

_
= (1, 0, 0, ..., 0)
1
2

1
a +
1
=
1
2

1
a =
1

( ) =
1
2
a
a = 2 ( )
This is identical with equation (I).
The system is separable.
Note:
_

_
2
+
d
2
sin
2

=
2

r
2
_

_
2
+
r
2
_

r
r
+
_ =
2

We all now assume that the system is separable.

r
r
=
_
2
_

r
+ dt
_

r
2

d
2
sin
2

which is immediatery integrable and nd


r
,

are already found out.


Hence the system is separable and hence reduces of quadratare

r
+

= 0.
Discuss the keplers problem using separability.
Let us consider a particle of unit mass p attached towards O by an
iverse square gravitational force.
Let T be the K.E. of the system.
Let V be the P.E. of the system.
153
T =
1
2
_
r
2
+ r
2

2
+ r
2

2
sin
2

_
dr =

r
L = T V =
1
2
_
r
2
+ r
2

2
+ r
2

2
sin
2

_
+

r
Now, we can nd generalized moments
p
r
=
L
r
=
1
2
2 r = r
p

=
L

=
1
2
r
2
2

= r
2

=
L

=
1
2
r
2
sin
2
2

= r
2
sin
2



=
p

r
2
sin
2

T =
1
2
_
p
2
r
+
p
2

r
2
+
p
2

r
2
sin
2

_
where p
r
, p

and p

are the generalized moments.


This system is an orthogonal system, we have H = T +V = dt.
The modied Hamilton Jacobi equation is
1
2
_

r
r
_
2
+
1
2r
2
_

_
2
+
1
2r
2
sin
2

_
1


r
= dt (1)
where =
r
(r) +

() +

() .
is missing from the above equation since is ignorable co-
ordinates..
[r() = d()] p

(1) 2r
2
r
2
_

r
r
_
2
2r
2
_

r
+
t
_
+
_

_
2
+
1
sin
2

_
2
= 0
The rst two terms are functions of r only and the last terms are
fuctions of .
154
Hence they are each to seperable constant.
r
2
_

r
r
_
2
2r
2
_

r
+
t
_
=
2
(2)
_

_
2
+
1
sin
2

_
2
=
2
(3)
(2) r
2
_

r
r
_
2
=
2

+ 2r
2
_

r
+
t
_
= 2r
2
_

r
+
t
_

r
r
=
_
2
_

r
+
t
_

r
2
(3)
_

_
2
=
2

_
2
1
sin
2

sin
2

which is immeadiately integrable and nd


r
,

are already found out.


Hence the system is separable and hence reduces to quadrative

r
+

= 0.
Jacobi Theorem :
If s (q, , t) is any complete solution of the Hamilton Jacobi equa-
tion
s
t
+ H
_
q,
s
q
, t
_
= 0,
i
=
s

i
, p
i
=
s
q
i
, i = 1, 2, 3, ..., n
where s are arbitrary constants are used to solve q
i
(, , t) and p
i
(, , t) . Then these expression provide the general solution of the
canonical equation associated with Hamilton H(q, p, t).
155
Proof :
s
t
+ H
_
q,
s
q
, t
_
= 0 (1)
Dierentiating (1) w.r.to
i
, by considering p
j
as a function of (q, , t)

2
s

i
t
+
n

j=1
H
p
j
p
j

i
= 0 (2)
Given
s

i
=
i
Dierentiating both sides w.r.to t

2
s
t
i
+
n

j=1

2
s
q
j

i
q
j
= 0 (3)
(3) (2)
n

j=1
q
j

2
s
q
j

H
p
i
p
i

i
= 0
p
i
=
s
q
j
Dierentiating w.r.to
i
,
p
i

i
=

2
s

i
q
j
Using this into second term (4) becomes
n

j=1
q
j

2
s
q
j

H
p
i

2
s

i
q
j
= 0
n

j=1
_
q
j

H
p
i
_

2
s

i
q
j
= 0.
Now

2
s

i
q
j

= 0
156
Each co-eceient must be in this
q =
H
p
j
.
This is Hamilton rst equation.
In order to derive the Hamilton second equation we again dieren-
tiate (1) partially w.r.to q
j
and associated p
i
= f (q, , t)

2
s
q
j
t
+
n

i=1
H
p
i
p
i
q
j
+
H
q
j
= 0 (5)
p
j
=
s
q
j
p
j
=

2
s
q
j
t
p
j


2
s
q
j
t

n

i=1

2
s
q
j
q
i
q
i
= 0 (6)
(5) + (6)
n

i=1
H
p
i
p
i
q
j
+
H
q
j
+ p
j

i=1

2
s
q
j
q
i
q
i
= 0 (7)
But
H
q
j
= q
j
and
p
i
q
j
=

2
s
q
i
q
j
First and last terms get called in (8) reduces to
H
q
j
+ p
j
= 0
p
j
=
H
q
j
This is second kind equation.
157
Unit-V
Canonical transformation
Let us consider only a holonomic system which can be described by
the standard form of either Hamiltons (or) Lagranges equation.
Hamiltons principle applies to these system the generalized coordi-
nates are q
1
, q
2
, . . . , q
n
. We know that the Hamilton principle is
I = 0 where I =
t
1
_
t
0
Ldt

t
1
_
t
0
L(q, q, t) dt = 0
where the endpoints of the varied paths are xed in both conguration
space and time.
Let L = T V.
Now if a new set of coordinates Q
1
, Q
2
, . . . , Q
n
is Q related to the
formal set by a point transformation. The Larangian function is given
by,
L

_
Q,

Q, t
_
= L(q, q, t) = T V
i.e., L and L

are equal. Now the Hamiltons principle function


I = 0 .

t
_
t
0
L

_
Q,

Q, t
_
dt = 0.
Now we consider the Lagrangian equation is
L

_
Q,

Q, t
_
= L(q, q, t)
d
dt
(q, Q, t)
158
where (q, Q, t) is twice dierential. Then,
I =
t
_
t
0
L

_
Q,

Q, t
_
dt
=
t
_
t
0
_
L(q, q, t)
d
dt
(q, Q, t)
_
dt
=
t
_
t
0
L(q, q, t) dt
t
_
t
0
d(q, Q, t)
=
t
_
t
0
L(q, q, t) dt [(q, Q, t)]
t
t
0
I =
t
_
t
0
L(q, q, t) dt
i.e., q s and Q s are zero at the xed time t
0
and t .

t
_
t
0
L

_
Q,

Q, t
_
= 0.
Now Hamilton are described by the equation
H (q, p, t) =
n

i=1
p
i
q
i
L(q, q, t)
K (Q, P, t) =
n

i=1
p
i

Q
i
L

_
Q,

Q, t
_
where the generalized moment are given by,
p
i
=
L
q
i
, P
i
=
L


Q
i
q
i
=
H
p
i
, p
i
=
H
q
i
, i = 1, 2, . . . , n.
159
Here the function L

_
Q,

Q, t
_
.
The Hamilton canonical function

Q
i
=
K
P
i
,

P
i
=
H
Q
i
, i = 1, 2, . . . , n
A transformation form (q, p) to (Q, P) which preserves the canon-
ical form of the equation of motion is known as canonical transforma-
tion.
Dierential forms of Pfaan dierential equation:
Let us consider a Hamilton function H (q, p, t) suppose the trans-
formation equation is of the form,
Q
i
= Q
i
(q, p, t) , P
i
= P
i
(q, p, t) i = 1, 2, . . . , n
where each function is atleast twice dierentiable. Now the Lagrangian
equation is given by,
L

_
Q,

Q, t
_
= L(q, q, t)
d
dt
[(q, Q, t)] (1)
Hamilton function is deed by,
H (q, p, t) =

p
i
q
i
L(q, q, t) (2)
K (Q, P, t) =

P
i

Q
i
L

_
Q,

Q, t
_
. (3)
Using (2) and (3) in (1) we get

P
i

Q
i
K (Q, P, t) =

p
i
q
i
H (q, p, t)
d
dt
[(q, Q, t)]

P
i
d
dt
(Q
i
) K =

p
i
d
dt
(q
i
) H
d
dt
()
d =

p
i
dq
i
Hdt

P
i
dQ
i
+ Kdt (4)
where is the generalized function. Equation (4) is called the dier-
ence of two pfaan dierential equation and (4) is an exact dierential
equation
160
Note: The function (q, Q, t) is called the generating function for
the transformation.
Book work: Consider Q =

2qe
t
cos p, P =

2qe
t
sin p .
Show that the above transformation is canonical.
Solution: Given that
Q =
_
2qe
t
cos p,
P =
_
2qe
t
sin p.
We obtained
pq PQ = (1)
Q = e
t
_
2q cos p
2

2q
q +
_
2q (sin p) p
_
= e
t
_
cos p

2q
q sin p
_
_
2q
_
p
_
(1)
pq PQ = pq
_
2qe
t
sin p
_
e
t
_
cos p

2q
q sin p
_
2qp
__
= pq sin p cos pq + 2q sin
2
pp
= [p sin p cos p] q + 2q sin
2
pp (2)
which is the dierential form,
pdx + qdy = f (x)
and
P
y
=
Q
x

P
p
=
Q
q

p
(p sin p cos p) = 1
_
sin p (sin p) + cos
2
p

= 1
_
sin
2
p + cos
2
p

= 1 cos 2p
161

p
[p sin p cos p] = 2 sin
2
p

p
[p sin p cos p] = 2 sin
2
p
Similarly,

q
_
2q sin
2
p

= 2 sin
2
p

p
[p sin p cos p] =

q
_
2q sin
2
p

The transformation is caonical.


Example: Let us consider the transformation
Q =
1
2
_
q
2
+ p
2
_
, P = tan
1
_
q
p
_
Show that the transformation is canonical. Find the new Hamilton
canonical equation.
Solution: Given
P = tan
1
_
q
p
_
;
Q =
1
2
_
q
2
+ p
2
_
.
We obtained
pq PQ = d (1)
Q =
1
2
2qq +
1
2
2pp
Q = qq + pp
(1)
pq pQ = pq + tan
1
_
q
p
_
(qq + pp)
162
= pq + tan
1
_
q
p
_
qq + tan
1
_
q
p
_
pp
=
_
p + q tan
1
_
q
p
__
q + p tan
1
_
q
p
_
p

p
_
p + q tan
1
_
q
p
__
= 1 +q
_
_
_
_
1
1 +
q
2
p
2
_
_
_
_
_

q
p
2
_
= 1 +q
_
p
2
p
2
+ q
2
__

q
p
2
_
= 1
q
2
p
2
+ q
2
=
p
2
+ q
2
q
2
p
2
+ q
2
=
p
2
p
2
+ q
2

p
_
p + tan
1
_
q
p
__
=
p
2
p
2
+ q
2

q
_
p + tan
1
_
q
p
__
= p
_

_
p
_
_
_
_
1
1 +
q
2
p
2
_
_
_
_
_

_
_
1
p
_
= p
_
p
2
p
2
+ q
2
__
1
p
_
=
p
2
p
2
+ q
2

p
_
p + tan
1
_
q
p
__
=

Q
_
p + tan
1
_
q
p
__
The transformation is canonical.
Principle forms of generating functions:
Let us consider a generating function (q, Q, t) . The other types
of generating functions namely F
2
(q, P, t) , F
3
(P, Q, t) and F
4
(p, P, t)
163
. Now we show the relationship of F
2
and F
1
and again F
3
and F
4
and
F
1
and F
4
.
The relationship of the generating function F
1
, F
2
, F
3
and F
4
are
called the principle of generating function.
Proof: Given that the generating function (q, Q, t) . We know
that the dierential form

p
i
q
i

P
i
Q
i
Hdt + Kdt = dF
1
(q, Q, t) . (1)
Let us consider

Q
i
dP
i

P
i
dQ
i
= d
_

Q
i
P
i
_
. (2)
Adding (1) and (2) we get

P
i
dq
i
+

Q
i
dP
i
Hdt + Kdt = dF
2
(q, P, t) (3)
where dF
2
(q, P, t) = dF
1
(q, Q, t) + d
_
n

i=1
Q
i
P
i
_
.
F
2
(q, P, t) = F
1
(q, Q, t) +

Q
i
P
i
. (4)
Equation (4) gives the relationship in between F
1
and F
2
. Now the
total dierential form of F
2
is the form
dF
2
=

F
2
q
i
dq
i
+

F
2
P
i
dP
i
+
F
2
t
dt. (5)
Equating the coecient in (3) and (5)
P
i
=
F
2
q
i
, Q
i
=
F
2
p
i
, K =
F
2
t
+ H. (6)
Equation (6) is called the canonical transformation of the function F
2
.
Similarly we can derive the following equation associated with
F
3
(p, Q, t) .
164
Consider
F
3
(p, Q, t) = F
1
(q, Q, t)

q
i
p
i
(7)
(7)
F
3
= dF
1
(q, Q, t)

q
i
dp
i

p
i
q
i
Using (1),
dF
3
=

p
i
dq
i

P
i
dQ
i
Hdt + Kdt

q
i
dp
i

p
i
dq
i
dF
3
( p, Q, t) =

P
i
dQ
i
Hdt + Kdt

q
i
dp
i
(8)
Then the total dierentiating dF
3
(p, Q, t)
dF
3
(p, Q, t) =

F
3
P
i
dp
i
+

F
3
Q
i
dQ
i
+
F
3
t
dt. (9)
Equating the coecients in (8) and (9),
q
i
=
F
3
P
i
(i = 1, 2, . . . , n)
P
i
=
F
3
Q
i
(i = 1, 2, . . . , n)
K = H +
F
3
t
_

_
(10)
which is the canonical transformation of equation (8).
Similarly for next generating function F
4
(p, P, t) we derived as fol-
lows.
F
4
(p, P, t) = F
2
(q, P, t)

q
i
P
i
(11)
dF
4
= dF
2

p
i
dq
i

q
i
dp
i
Using (3),
dF
4
=

p
i
dq
i
+

Q
i
dP
i
Hdt +Kdt

p
i
dq
i

q
i
dp
i
(12)
dF
4
=

Q
i
dP
i
Hdt + Kdt

q
i
dp
i
.
165
Taking the total dierential of F
4
,
dF
4
=

F
4
p
i
dp
i
+

F
4
P
i
dP
i
+
F
4
t
dt (13)
Equating the coecient in (12) and (13),
Q
i
=
F
4
P
i
(i = 1, 2, . . . , n)
q
i
=
F
4
p
i
(i = 1, 2, . . . , n)
K = H +
F
4
t
_

_
(14)
which is the canonical transformation of F
4
(p, P, t) .
Equations (4), (7) and (11) gives the relation between the generat-
ing function which is the principle function.
Book Work: Consider the transformation
Q = log
sin p
q
, P = q cot p
Obtain the four major types of generating function associated with
the transformation.
Proof: Given that,
Q = log
sin p
q
, P = q cot p
We know that,
pq PQ = (1)
Q =
1
sin p
q
_
1
q
cos pP + sin p
_

1
q
2
_
q
_
=
q
sin p
_
cos p
q
P
sin p
q
2
q
_
= cot pp
1
q
q
166
pq q cot p
_
cot pp
1
q
q
_
=
pq q cot
2
pp + cot pq =
(p + cot p) q q cot
2
pp =

p
(p + cot p) = 1 +
_
cosec
2
p
_
= 1 cosec c
2
p = cot
2
p

p
(p + cot p) = cot
2
p

q
_
q cot
2
p
_
= cot
2
p

p
(p + cot p) =

q
_
q cot
2
p
_
=
The transformation is canonical.
Let = pq + q cot p (from (1)).
The geometrical representation of the transformation
sin p =
q
e
Q
= qe
Q
cos p =
p
e
Q
= pe
Q
cot p =
cos p
sin p
=
pe
Q
qe
Q
=
p
q
q cot p = p
Now,
p
2
+ q
2
=
_
e
Q
_
2
= e
2Q
p
2
= e
2Q
q
2
p =
_
e
2Q
q
2
167
=
1
e
Q
_
_
1 q
2
e
2Q
_
We know that
cos p = pe
Q
=
1
e
Q
_
_
1 q
2
e
2Q
_
e
Q
cos p =
_
1 q
2
e
2Q
p = cos
1
_
_
1 q
2
e
2Q
_
From the principle of generating function
(p, q) = F
1
(q, Q) (2)
F
1
(q, Q) = q cos
1
_
_
1 q
2
e
2Q
_
+
_
e
2Q
q
2
(3)
F
1
q
= cos
1
_
_
1 q
2
e
2Q
_
= p
F
1
Q
=
_
e
2Q
q
2
= p
Comparing (2) and (3) equating the coecient we get
p = cos
1
_
_
1 q
2
e
2Q
_
,
p =
_
e
2Q
q
2
.
The above canonical can be written as
F
1
q
= cos
1
_
_
1 q
2
e
2Q
_
= p,
F
1
q
=
_
_
e
2Q
q
2
_
= p.
168
To nd the generating function F
2
(q, P) . From the principle of gen-
erating function
F
2
(q, P) = F
1
(q, Q, t) +

Q
i
P
i
F
2
= pq + q cot P + QP (4)
F
2
is the generations function of the variable (q, P) .
We know that
q cot p = P
cot p =
P
q
tan p =
q
P
p = tan
1
_
q
P
_
From the triangle
P
2
= e
2Q
q
2
e
Q
=
_
p
2
+ q
2
e
Q
=
1
_
p
2
+ q
2
Q = log
_
1
_
p
2
+ q
2
_
= log 1 log
_
_
p
2
+ q
2
_
= log
_
_
p
2
+ q
2
_
Substitute p, Q in equation (4) we get
F
2
(q, P) = q tan
1
_
q
P
_
+ p +
_
log
_
_
p
2
+ q
2
__
p
= q tan
1
_
q
P
_
+ p
_
1 log
_
_
p
2
+ q
2
__
(5)
From (4) and (5) we get
169
F
2
q
= tan
1
_
q
P
_
= p
F
2
p
= 0 log
_
q
2
+ p
2
= Q
which are the canonical equation of the function F
2
.
To nd the function F
3
,
F
3
= F
3
(p, Q)
We know that,
F
3
(p, Q) = F
1
(q, Q)

q
i
p
i
= qp + q cot P qp
F
3
= q cot p (6)
q cot p = q
cos p
sin p
= q
Pe
Q
qe
Q
= p
Next to nd the function F
4
(p, P) .
Similarly
F
4
(p, P) = F
2
qp
= qp + q cot p + QP qp
= q cot p + QP
F
4
(p, P) = P + QP
cos p = p = Pe
Q
e
Q
=
cos P
p
Q = log
_
cos P
p
_
F
4
= p + log
_
cos P
p
_
P
170
F
4
= p + P log
_
cos p
p
_
F
4
p
= q (tan p cot p) = p tan p = q
F
4
p
= log
_
cos P
p
_
= Q
Hence the four generating function.
Special transformation
1.Identity transformation:
Let us consider the identity transformation which is an obvious
example of a canonical transformation. It is generated by a function
is of the form,
T
n
=
n

i=1
q
i
p
i
(1)
Then
P
i
=
F
2
q
i
= p
i
(i = 1, 2, . . . , n)
Q
i
=
F
2
P
i
= q
i
(i = 1, 2, . . . , n) . (2)
The identity transformation is generated by
F
3
=
n

i=1
P
i
Q
i
.
2. Orthogonal transformation:
This is the simple canonical transformation. The orthogonal trans-
formation of q s and p s generated by
F
2
=
n

i=1
n

j=1
a
ij
p
i
q
j
,
where a
i
s are constant meeting the orthogonality conditions
n

i=1
a
ij
a
jk
=
jk
.
171

jk
is the kroneckal delta the orthogonality can be written in the matrix
form
a a
1
= a
1
a = 1.
The transformation equation are
P
j
=
F
2
q
i
=

a
ij
, Q
i
=
F
2
p
i
=

a
ij
q
j
.
These equation can be solved for p
i
s in terms of P
i
s and Q
i
s in
terms of q
i
s then the transformation equations are
Q
i
=
n

j=1
a
ij
q
i
P
i
=
n

j=1
a
ij
P
j
(i = 1, 2, . . . , n)
These equations represents the equal rotation is q -space and p -space.
3. Translation:
We know that F
2
=

q
i
P
i
, replace q
i
by q
i
+c
i
and replace P
i
by
P
i
d
i
F
2
=

q
i
P
i
=

(q
i
+ c
i
) (P
i
d
i
)
=

q
i
p
i
+ c
i
P
i
d
i
We have to omit c
i
d
i
. Now,
p
i
=
F
2
q
i
= P
i
d
i
Q
i
=
F
2
P
i
= q
i
+ c
i
Q
i
= q
i
+ c
i
,
P
i
= P
i
+ d
i
(i = 1, 2, . . . , n)
172
which represents the required translation.
Homogeneous Canonical transformation:
We know that the dierential form is
n

i=1
p
i
q
i

i=1
P
i
Q
i
= (1)
is an exact dierential. Consider the function and are identically
zero. Then
n

i=1
(P
i
q
i
P
i
Q
i
) = 0
and the corresponding transformation is called a homogeneous canon-
ical transformation. This transformation is also known as Mathieu
transformation (or) contact transformation.
Point transformation:
We consider the class of homogeneous canonical transformation for
which full set of n independent function
j
(q, Q, t) exist and equal to
zero.
This implies that the two n n coecient matrices of
n

i=1

j
q
j
q
i
+
n

i=1

j
Q
i
Q
i
= 0 (j = 1, 2, . . . , m)
are non singular and the corresponding determinants are non zero,
that is,

j
q
i

= 0,

j
Q
i

= 0.
Hence we can solve for the Qs in terms of (q, t) on for the q s as
functions of (Q, t) .
We have n equation is of the form
Q
i
= f
i
(q, t) i = 1, 2, . . . , n
where the f
i
s are twice dierentiable. These equations represent
a point transformation. i.e., they represent a mapping of points in
173
conguration space. If one also includes the equations for the P s,
the entire set represents a transformation in phase space and is known
as extended point transformation.
Momentum transformation:
The momentum transformation equation are obtained from
p
i
=
n

j=1
P
j
Q
i
q
i
(i = 1, 2, . . . , n)
which in this case takes the form
p
i
=
n

j=1
f
i
q
i
(i = 1, 2, . . . , n) .
Thus the p s are homogeneous linear functions of the P s and vice
versa. If we dene n s of the form

j
= Q
j
f
j
(q, t) (j = 1, 2, . . . , n)
If the represents the transformation is called the momentum transfor-
mation.
Non homogeneous point transformation:
Let the basic dierential form be
n

i=1
p
i
q
i

i=1
P
i
Q
i
= . (1)
Consider
Q
i
= f
i
(q, t) ,
Q
j
= f
j
(q, t)
Q
j
=
n

j=1
f
i
q
i
q
i
. (2)
174
Substitute the equation (2) in (1) implies
n

i=1
p
i
q
i

i=1
P
i
_
n

j=1
f
i
q
i
q
i
_
= . (3)
In general,
=
n

i=1

q
i
q
i
+
n

i=1

P
i
p
i
(4)
Here the q s and p s are independent.Equation (3) and (4) we get

q
i
=
n

i=1
p
i

i=1
P
i
n

j=1
f
j
q
i
=
n

i=1
_
p
i
P
i
n

j=1
f
j
q
i
_
p
i
=

q
i
+ P
i
n

j=1
f
i
q
i
(i = 1, 2, . . . , n)


p
i
= 0
Equation (4) implies (q, t) is not an explicit function of the p s.
Furthermore it has the proper form for a generating function of the
type of f
i
. Hence we can choose F
1
to be identically equal to for a
point transformation.
Example: Illustrate the theory for the nonhomogeneous case, sup-
pose we are given sceleronomic extended point transformation
Q = tan q, P = (p m
0
) cos
2
q,
where m and
0
are constants.
Solution: Given that Q = tan q, P = (p m
0
) cos
2
q , we wish
to show that the transformation is canonical.
Q = sec
2
qq
= Pq =
_
(p m
0
) sec
2
q

sec
2
qq
175
=
pq = pq + m
0
q =
m
0
q = (1)
which is an exact dierential integrating
F
1
= = m
0
q. (2)
Hence this is a nonhomogeneous canonical transformation. Now the
point transformation is,
Q
i
(q, t) = f
i
(q, t)
Q = tan q f (q) .
From the non homogeneous point transformation

j
= Q
i
f
i
(q, t)
= Qtan q
which yields K = H . Thus the two Hamilton functions are equal
F
2
(q, P) = F
1
+ QP
= mv
0
q + QP
= mv
0
q + tan qP
p =
F
2
q
= mv
0
+ P sec
2
q
Q =
F
2
P
= tan q.
Here the canonical equations for the generating functionF
2
.
To nd F
3
:
F
3
(P, Q) = F
1
qP
176
= mv
0
q qP
F
3
P
= q,
F
3
Q
=
mv
0
1 +Q
2
.
To nd F
4
:
F
4
(p, P) = F
2
qp = F
1
+ QP qp
= mv
0
q + tan qP qp
F
4
p
= q,
F
4
P
= tan q.
Now let us apply this transformation to a specic physical situation.
Consider a mass spring system which is attached to a plane that is
translating with a constant velocity v
0
as shown in the following gure.
0
v
k
m
0
q + l
The unstressed length of the spring. Hence we have a conservative
holonomic system (or) Hamiltons equations.
L = T V
L =
1
2
m(v
0
+ q)
2

1
2
Kq
2
p =
L
q
=
1
2
m2 (v
0
+ q) = m(v
0
+ q)
The Hamilton function is H = T
2
T
0
+ V from the mass spring
system.
H =
p
2
2m
v
0
p +
1
2
Kq
2
177
Hence the canonical equation of the motion for (q, p) variables are,
q =
H
p
=
p
m
v
0
,
p =
H
q
=
1
2
K (2q) = Kq
Now let us obtain L

_
Q,

Q
_
relative to the moving frame. Here,
T =
1
2
m q
2
V =
1
2
Kq
2
Given that Q = tan q q = tan
1
Q.
q =
1
1 +Q
2

Q
L

= T V
=
1
2
m
_

Q
2
(1 + Q
2
)
2
_

1
2
K
_
tan
1
Q
_
2
=
1
2
m

Q
2
(1 + Q
2
)
2

1
2
K
_
tan
1
Q
_
2
P =
L


Q
=
m2

Q
2 (1 +Q
2
)
2
=
m

Q
(1 + Q
2
)
2
Given that
P = (p mv
0
) cos
2
q
cos
2
q =
P
p mv
0
=
m

Q
(1 +Q
2
)
2
m q
=

Q
q (1 + Q
2
)
2
cos
2
q =
Q
q
(1 +Q
2
)
2
=
1 + Q
2
(1 + Q
2
)
2
=
1
1 + Q
2
_
Since

Q
q
= 1 + Q
2
_
178
cos q =
1
_
1 + Q
2
In this case in K.E consist of T
2
terms only Hamiltonian function.
H = T
2
+ V
=
p
2
2m
+
1
2
Kq
2
K = T + V
=
1
2
m q
2
+
1
2
Kq
2
=
1
2
m
_

Q
2
(1 + Q
2
)
2
_
+
1
2
K
_
tan
1
Q
_
2
We know that
P =
m

Q
(1 + Q
2
)
2
m

Q = P
_
1 +Q
2
_
2

Q =
P (1 + Q
2
)
2
m
K =
1
2
m

Q

Q
(1 +Q
2
)
2
+
1
2
K
_
tan
1
Q
_
2
=
1
2
p
(1 + Q
2
)
2
(1 + Q
2
)
2
p
(1 +Q
2
)
2
m
+
1
2
K
_
tan
1
Q
_
2
=
p
2
2m
_
1 + Q
2
_
2
+
1
2
K
_
tan
1
Q
_
2
Here the canonical equations are

Q =
K
p
=
1
2m
2p
_
1 + Q
2
_
2
=
p
m
_
1 +Q
2
_
2

P =
K
Q
=
_
p
2
2m
__
1 + Q
2
_
2

+
1
2
K2 tan
1
Q
_
1
1 + Q
2
__
179
=
2p
2
Q
m
_
1 + Q
2
_

K tan
1
Q
1 + Q
2
.
Hence the canonical equation.
Lagrangian and Poisson brackets:
Lagrangian brackets:
Suppose the given transformation equation
Q
i
= Q
i
(q, P, t)
P
i
= P
i
(q, p, t)
(i = 1, 2, , n)
Let us consider the canonical dierential form
n

i=1
p
i
q
i

i=1
P
i
Q
i
= (1)
Q
i
=
n

i=1
Q
i
q
i
q
i
+
n

i=1
Q
i
P
j
P
j
(2)
Using (2) in (1)
n

i=1
p
i
q
i

i=1
P
i
_
n

j=1
Q
i
q
j
q
j
+
n

j=1
Q
i
P
j
P
j
_
=
n

j=1
_
p
j

i=1
P
i
Q
i
q
j
_
q
j

i=1
n

j=1
P
i
Q
i
P
j
P
j
=
Now,

q
k
_
p
j

i=1
P
i
Q
i
q
i
_
=

q
j
_
p
k

i=1
P
i
Q
i
q
k
_

p
k
_
n

i=1
P
i
Q
i
p
j
_
=

p
j
_
n

i=1
P
i
Q
i
P
k
_

p
k
_
p
j

i=1
P
i
Q
i
q
j
_
=

q
j
_

i=1
P
i
Q
i
p
k
_
180
where j, k = 1, 2, . . . .n.These equation can be written as
n

i=1
_
Q
i
q
i
P
i
q
k

P
i
q
i
Q
i
q
k
_
= 0
n

i=1
_
Q
i
p
j
P
i
p
K

P
i
p
j
Q
i
p
K
_
= 0
n

i=1
_
Q
i
q
j
P
i
p
K

P
i
q
j
Q
i
p
K
_
=
jk
where
jk
is the kronecker delta.
Denition: Lagrangian brackets expression of two variable (u, v)
by using the notation
[u, v] =
n

i=1
_
Q
i
u
P
i
v

P
i
u
Q
i
v
_
where u andv are any two variables q
1
, q
2
, . . . , q
n
, p
1
, p
2
, . . . , p
n
.
Skew symmetry: As a consequence of a two skew symmetry of
the Lagrangian bracket is
[u, v] = [v, u]
and
[u, u] = [v, v] = 0.
Invariant: A general characteristic of the Lagrangian bracket is
invariant under a canonical transformation.
Example: If the 2n variables (q, p) and in variable (Q, P) are
related by a canonical transformation. The either set may be used in
equating a given bracket evaluation. Thus we have
[u, v] =
n

i=1
_
q
i
u
p
i
v

p
i
u
q
i
v
_
=
n

i=1
_
Q
i
u
P
i
v

P
i
u
Q
i
v
_
181
where each set of dynamical variable in assume to be a function of
(u, v) as well as other variable.
Poisson brackets: Suppose that the function of the dynamical
variable and time namely u = u(q, p, t) and v = v (q, p, t). The poisson
bracket expression for function is
(u, v) =
n

i=1
_
u
q
i
v
p
i

u
p
i
v
q
i
_
.
In the case of Lagrangian brackets, we have
(u, v) = (v, u)
(u, u) = (v, v) = 0.
The poisson brackets is useful testing whether the transformation is
canonical (or) not.
Poisson theorem:
Statement: If u(q, p) and v (q, p) are integrals of a Hamiltonian
system, then the Poisson bracket (u, v) is also an integral, that is (u, v)
is constant of the motion.
Proof: Let u and v be integrals of the motion. We know that,
df
dt
= (f, H) +
f
t
,
(u, H) +
u
t
= 0, (1)
(v, H) +
v
t
= 0. (2)
Also,
d
dt
(u, v) = [(u, v) , H] +

t
(u, v)
d
dt
(u, v) = [(u, v) , H] +
_
u
t
+ v
_
+
_
u,
v
t
_
(3)
182
From (1) and (3)
d
dt
(u, v) = [(u, v) , H] [(u, H) , v] [u, (v, H)]
= [(u, 0) , H] + [(H, u) , v] + [(v, H) , u]
By Jacobi identity,
[u, (v, w)] + [v, (w, u)] + [w, (u, v)] = 0

d
dt
(u, v) = 0.
Hence the proof.
Example: Consider the transformation
Q =
_
e
2q
p
2
, P = cos
1
(pe
q
) .
Use the poisson bracket to show that it is canonical.
Solution: We know that the poisson bracket
(u, v) =
n

i=1
_
u
q
i
v
p
i

u
p
i
v
q
i
_
. (1)
Given that
Q =
_
e
2q
p
2
,
P = cos
1
(pe
q
) .
Here u = Q, v = P.
(Q, P) =
Q
q
P
p

Q
q
P
p
(2)
Q =
_
e
2q
p
2
Q
q
=
1
2
_
e
2q
(2)
_
e
2q
p
2
_
183
=
e
2q
_
e
2q
p
2
=
p
_
e
2q
p
2
P = cos
1
(pe
q
)
P
q
=
(1) pe
q
_
1 p
2
e
2q
=
p
_
e
2q
p
2
P
p
=
e
2q
_
1 p
2
e
2q
=
1
_
e
2q
p
2
.
Using these values in (2) we get
(Q, P) =
e
2q
_
_
e
2q
p
2
_
2

p
2
_
_
e
2q
p
2
_
2
=
e
2q
p
2
e
2q
p
2
= 1
We know that (u, u) = (v, v) = 0
(Q, Q) = 0, (P, P) = 0
Hence the given transformation is canonical.
Bilinear co-variant:
This is the another method which may be used in testing whether
a given transformation is canonical involves the bilinear co-variant.
Suppose we consider the Pfaan dierential
=
n

i=1
X
i
(x) dx
i
(1)
where the x s are an independent set of innitesimal displacements
from the same reference point. Now we can write,
=
n

i=1
(X
i
dx
i
+ X
i
dx
i
)
184
dQ =
n

j=1
(dX
j
x
j
+ X
j
dx
j
)
where
X
i
=
n

j=1
X
j
x
j
x
j
,
dX
j
=
n

i=1
X
j
x
i
dx
i
.
Then we obtain
dQ =
n

i=1
n

j=1
_
X
i
x
j

X
j
x
i
_
dx
i
x
j
=
n

i=1
n

j=1
[(X
j
dx
i
dx
j
x
j
) + (X
j
dx
i
Xx
j
)] (2)
note that dx
i
= dx
i
Now, considering contemporaneous variants where

ij
=
X
i
x
j

X
j
x
i
.
We nd that
d =
n

i=1
n

j=1

ij
dX
i
x
j
This is the bilinear co-variant associated with the dierential equation
(1).
185

You might also like